Instructor’s Guide for ICD-9-CM Diagnostic Coding and ...



Basic Current Procedural Terminology and HCPCS Coding

2013 Edition

Instructor’s Manual

[pic]

Notes to Instructors

This Instructor Guide includes resources for instructors. This item and PowerPoint slides are available at . Chapter lesson plans include the several of the following elements:

• Bullet list summary of lesson focus

• Objectives

• Suggested student activities and assignments

• Key points for lecture notes

• Websites for faculty and/or students

• Additional Exercises with answer keys provided to faculty. Exercises include: practice fill-in-the-blank questions, CPT Assistant exercises and Test Bank Items

• Answer Key to Appendix C is provided (answers not provided to students)

The websites listed in this book were current and valid as of the date of publication. However, webpage addresses and the information on them may change or disappear at any time and for any number of reasons. The user is encouraged to perform his or her own general web searches to locate any site addresses listed here that are no longer valid.

Please note that a complete list of Web Resources is located in Appendix A. For an introduction, faculty may consider demonstrating to the student useful sites, such as:



Introduce students to the Communities of Practice (coding) and HIM Body of Knowledge.

Contents

Lesson Plans (including Objectives, Suggested Student Activities and Assignments, Key Points for Lecture Notes, and Test Bank Questions)

Chapter 1 4

Chapter 2 8

Chapter 3 14

Chapter 4 18

Chapters 5, 6, 8, and 9 32

Chapter 7 38

Chapter 10 43

Chapter 11 46

Key to Appendix C Additional Practice Exercises (Operative Reports) 53

Answers to Chapter Review Exercises, Appendix D 57

Chapter 1

Introduction to CPT and HCPCS

This chapter will focus on the following topics:

• Introduction to CPT and HCPCS

• HCPCS Coding

• Claim forms (CMS-1500 and CMS-1450)

Objectives

After completing this chapter, the student should be able to do the following:

1. Define coding

2. Discuss the purpose and future of coding

3. Identify the purpose and uses of CPT

4. State the official publication for CPT coding

5. Define and list the components of HCPCS

6. Distinguish between CPT and National Codes

7. Describe the general principles of medical record documentation

8. Given a list of services, identify what cannot be coded with CPT

9. Compare and contrast CPT with ICD-9-CM

10. Identify the ICD-9-CM/HCPCS coding assignment necessary for physician services, hospital inpatients, and hospital ambulatory patients

11. Differentiate between CMS-1500 and UB-04 forms

12. Define key words

13. Identify abbreviations

Suggested Student Activities and Assignments

1. Read current articles about current trends in coding (e.g. Computer Assisted Coding). Write a reaction paper and/or summary of the article.

2. Go to CMS’ website to review resources (e.g. HCPCS online file of codes, Medicare Coverage Determination).

3. Review AMA’s website for updates for CPT Category II and III codes.

4. Review websites dedicated to coding (see Web Resources located in Appendix A of the textbook). Summarize each site and/or rate the site’s usefulness.

5. During one of the first lectures, walk the students through the entire coding process.

Suggestion for Faculty

Refer to the AHIMA Practice Briefs and Audio Seminars to supplement lectures with advanced content concepts.

Introduce students to web resources.

Key Points for Lecture Notes

1. The American Medical Association (AMA) and some third-party payers developed CPT in the 1960s. It was used as a communication tool between third-party payers and physicians. Unlike ICD-9-CM (and ICD-10-CM), CPT was developed for reimbursement purposes.

2. Point out the differences between ICD-9-CM and CPT. Take some time to preview each of the code books and note the characteristics. Announce that ICD-10-CM is replacing ICD-9-CM in October 2014 and the plan for transitioning to the new version of the classification system. Differentiate between ICD-10-PCS and CPT.

3. It was not until the mid-1980s that hospitals began to use CPT. The Omnibus Budget Reconciliation Act demanded that hospitals use CPT to report outpatient services.

4. In the textbook, you will find examples of both UB-04 and CMS-1500 forms. Explain that claim forms are electronically submitted. CMS-1500 forms are used only in physician offices. UB-04 is the billing form for all hospital services: inpatient, outpatient, emergency, clinic, and so on. Introduce HIPAA 5010 electronic transaction standards and how it paves the way for ICD-10-CM/PCS.

5. Some of the routine services, in the hospital, are contained in a computerized file called a chargemaster. For example, consider a patient seen as an outpatient for a chest X-ray. Almost all facilities have the CPT codes for chest X-ray in the computer file; therefore, coders rarely assign these codes.

6. HCPCS is the most confusing part of the unit. When CMS decided to use CPT for reimbursement to hospitals/physician offices, there was one big hurdle: Not everything that is reimbursed by Medicare/Medicaid is contained in CPT (for example: drugs, supplies, and ambulance services). Keep in mind that CMS does not own CPT; therefore, CMS cannot control the contents. To supplement CPT, CMS wrote its own book of codes called Level II National Codes. When billing Medicare for an ace bandage given to a patient, the coder would have to reference Level II codes, because there are no codes in CPT for supplies.

7. Provide a list of services not in CPT.

8. CMS subcontracts with local carriers (now called MACs- Medicare Administrative Contractors) to manage claims processing in the geographic area. Prior to 2004, CMS gave local carriers the authority to also create codes, if the need arose. These codes were called Level III or Local Codes. The codes were communicated through newsletters called Medicare Bulletins. This two-part system is called HCPCS. Sometimes CPT and HCPCS are used interchangeably but that technically is not correct.

9. Call attention to the fact that CPT/HCPCS is used to identify procedure codes, and ICD-9-CM continues to be the coding system for all healthcare providers for diagnosis codes. ICD-9-CM describes “why” services were performed and CPT/HCPCS can be describes “what” services were performed. ICD-9-CM to be the only system for inpatients and will continue with implementation of ICD-10-CM/PCS in October 2014. Although third-party payers want CPT/HCPCS for billing purposes on outpatients, some hospitals continue to assign ICD-9-CM procedure codes for their hospitals’ database and for statistical reasons.

10. Many private insurers also accept HCPCS Level II codes.

Chapter 1 Test Bank Items

Instructions: Choose the best answer for each of the following questions.

1. Which of the following groups/organizations is responsible for maintaining the HCPCS Level II codes?

A. American Medical Association

B. American Hospital Association

C. Centers for Medicare and Medicaid Services

D. Local Fiscal Intermediary

2. The patient visits the physician’s office for a sore throat. The services for this patient would be submitted on what claim form?

A. HCPCS

B. UB-04

C. CMS-1500

D. CMS-1450

3. The patient is seen in the clinic for a laceration of the knee. The wound required suturing. On the claim form, which of the following types of codes would be assigned to represent the laceration?

A. ICD-9-CM

B. CPT

C. National Codes

D. Level III Codes

4. HCPCS was developed by:

A. American Medical Association

B. Department of Health and Human Services

C. Centers for Medicare and Medicaid Services

D. American Hospital Association

5. The CPT manual is published and maintained by:

A. American Medical Association

B. Department of Health and Human Services

C. Centers for Medicare and Medicaid Services

D. American Hospital Association

6. What codes will the hospital use on its billing form to present the diagnosis of “fractured humerus”?

A. CPT

B. HCPCS

C. ICD-9-CM

D. DSM-IV

7. For the December 7, 2012 patient encounter, the hospital will submit codes on what billing form?

A. UB-04

B. HCFA-1500

C. CMS-1500

D. UHDDS

8. The CPT book is updated:

A. As needed

B. For use in January

C. Monthly

D. For use in October

9. Which of the following procedures/services could not be assigned a code with CPT?

A. Gastroscopy

B. Anesthesia

C. Glucose tolerance test

D. Crutches

10. Which of the following can be identified as a National Code?

A. 36491

B. 4003F

C. L3381

D. 0030T

Chapter 2

Procedural Terminology in Current Use

This chapter will focus on the following topics:

• Format and organization of CPT

• Conventions and characteristics of CPT

• Alphabetic Index

• Abstracting documentation

• Coding references

Objectives

After completing this chapter, the student should be able to do the following:

1. Specify the conditions that must be met before a procedure or service is included in the CPT manual

2. Describe the contents of CPT: sections, subsections, subcategory heading, procedure, appendices, and index

3. Interpret conventions and characteristics of CPT:

a. Semicolon

b. Bullet

c. Triangle

d. Facing triangles

e. Plus sign

f. Null Symbol

g. Circled bullet

h. Pending FDA Approval Symbol

i. Resequenced Codes

j. Reinstated Code

4. Describe Category I, II, and III CPT Codes

5. Successfully apply the general rules/guidelines for coding assignment

6. Define key concepts:

a. Unlisted procedures

b. “See”

c. Stand alone

7. Given an operative report, successfully abstract pertinent clinical information

8. Reference official coding guidelines (CPT Assistant) to support accurate coding assignment

Suggested Student Assignments and Activities

1. Allow the students to work in groups and interpret documentation in the operative report; ask students to identify the main procedure.

2. Administer a self-assessment medical terminology quiz.

3. Look up and discuss unfamiliar terminology from operative reports.

4. Discuss the importance of using reference materials such as anatomical diagrams, surgical descriptions and CPT Assistant to support accurate coding assignments.

5. Research coding advice from CPT Assistant.

6. Access AMA’s website and discuss the uses of Category II and Category III codes.

7. Discuss articles from Body of Knowledge, for example:

• “Where to Find Answers to Coding Questions,” Journal of AHIMA (Coding Notes), April 2, 2010

• “Internet Resources for Accurate Coding and Reimbursement Practices,” AHIMA Practice Brief, March 31, 2010

• “Anatomy of a Physician Coder: Small, medium and large physician practices all feature these multi-tasking, multi-responsibility coders,” Journal of AHIMA, November 2, 2012.

Key Points for Lecture Notes

The textbook provides a good guide for the lecture in this unit.

1. Semicolon. This is one of the most confusing format concepts. If a code description contains a semicolon and there is one or more code description indented underneath, then the description before the semicolon is a “home description,” and the indented code needs the “home description” to fully complete the code description.

Note the second example: Osteotomy; tibia. Code 27705 should read: Osteotomy; tibia. Code 27707 should read: Osteotomy, fibula. The “home description” before the semicolon completes the description. It is just a way to save printing space.

Look at another example: 30150:

30150 Rhinectomy; partial

30160 total

If the physician documented that the patient had a total rhinectomy, the correct code would be 30160. Ask the students if they would assign CPT codes 30150 and 30160 together? Answer: No, it does not make sense. Either it is partial or total, not both. This is a good time to introduce students to the term “mutually exclusive.”

2. Category II and Category III Codes: AMA continues to expand the use of codes.

3. Alphabetic Index. The following are tips that are not in the textbook:

a. The index is more “free form” than ICD-9-CM. In ICD-9-CM, the index gives you reliable choices, but not always in CPT. The tabular codes rule the coding process.

b. After locating the code choice(s), ask the students to make a list to check in the tabular. These codes are tentative and need to be explored fully. Now ask the students to use process of elimination to choose the correct code. Students may have to use reference materials to help support their answer.

c. Read, read, and read again. Sometimes two codes look the same, so try and find out the difference. Encourage students to look up unfamiliar medical terms or abbreviations.

4. Coding operative reports. The textbook provides help for abstracting documentation from operative reports for coding. One of the most common mistakes for new coders is to code everything. Emphasize to student that they should not automatically assign codes for approaches and closures. Scan the operative reports for “action” words. Typical action words would be: excision, incision, endoscopy, exploration, etc.

5. Coding References. Emphasize the importance of using this reference to support their answers. To stress the importance of using references, two questions in the test bank items require students to reference a medical dictionary (questions 10 and 11).

6. AHIMA Coding Community and Body of Knowledge

Ask students to explore the CoPs and Body of Knowledge on AHIMA’s website. CodeWrite Community Newsletter is published monthly.

Chapter 2 Test Bank Items

1. Which of the following can be identified as a new code in CPT 2013?

a. 27065

b. 43252

c. 33218

d. 50593

2. Which of the following contains a comprehensive summary of CPT additions, deletions, and revisions since last year?

a. Appendix A

b. Appendix B

c. Appendix D

d. Appendix F

3. Reference 11920–19222 for tattooing. What is the correct code(s) for tattooing of 40 sq cm of skin?

a. 11920, 11921, 11922

b. 11921, 11922

c. 11922, 11922

d. 11920, 11922, 11922

4. Reference codes 49491-49525 for inguinal hernia repair. What is the correct code for an initial inguinal herniorrhaphy for incarcerated hernia (patient is 47 years old)?

a. 49496

b. 49501

c. 49507

d. 49521

5. Reference codes 31360-31368 for laryngectomy. What is the correct code assignment for a laryngectomy with subtotal supraglottic and radical neck dissection?

a. 31360, 31368

b. 31368

c. 31367

d. 31360

6. Reference codes 11200-11201 for removal of skin tags. What is the correct code(s) for removal of 16 skin tags?

a. 11200, 11200

b. 11200 x 16

c. 11200, 11201

d. 11201 x 16

7. Which of the following can be identified as an Anesthesia CPT code number?

a. 00300

b. 28270

c. 88304

d. 97035

8. Which of the following contains a complete description of CPT modifiers?

a. Appendix A

b. Appendix B

c. Appendix C

d. Appendix D

9. The surgeon performed a procedure that is unfamiliar to the coder and he or she is having trouble locating an appropriate CPT code. What should be the coder’s next action?

a. Assign an “unlisted procedure” code

b. Call the insurance company

c. Research the description about the procedure

d. Call the surgeon’s office

10. What is the correct code assignment for: destruction of 2 groups of internal hemorrhoids with use of infrared coagulation?

a. 46250

b. 46930

c. 46260

d. 46946

Practice Exercises

Assign the correct CPT surgical codes to the following. Do NOT append modifiers for this exercise.

1. Cystourethroscopy with biopsy 52204

2. Simple anterior nasal packing for epistaxis 30901

3. Blepharoplasty of the upper eyelid 15822

4. Incision and drainage of suppurative hidradenitis of arm 10060

5. Closed treatment of two rib fractures 21800, 21800

6. Insertion of temporary prostatic urethral stent 53855

7. Gait training, 15 minutes 97116

8. Diagnostic colonoscopy 45378

9. Lab test: Rubella antibody 86762

10. Excision of vagina papilloma 57135

11. Drainage of abscess of helix 69000

12. Operative Report

Procedure: Esophagogastroduodenoscopy

Esophagus: This area was well visualized in its entirety and appeared macroscopically normal. A 1.5 to 2-cm hiatus hernia was present. No macroscopic abnormalities were noted in the distal esophagus. A biopsy was taken approximately 5-cm proximal to the macroscopic gastroesophageal junction.

Stomach: The antrum was normal. On retroflexion, the cardia was minimally incompetent. The body of the stomach was normal. Biopsies were taken from the antrum and body of the stomach.

Duodenum: There was a moderate degree of duodenitis in the bulb. The postbulbar and descending duodenal areas appeared normal. A biopsy was taken from the bulb.

43239 Esophagoscopy with biopsy (supportive documentation: “biopsy was taken approximately 5-cm proximal…”)

CPT Assistant Exercise

Directions: Ask the students to answer these questions after referencing CPT Assistant.

1. A physician performed two trigger point injections in two different muscles. Would it be appropriate to report code 20552 twice for the two injections?

No. Codes 20552-20553 are reported one time per session, regardless of the number of injections or muscles injected

(CPT Assistant, May 2003, page 19).

2. The surgeon performs a laparoscopic appendectomy at the same time as the laparoscopic colostomy. Is it appropriate to report both the laparoscopic, appendectomy (code 44970) and the laparoscopic colostomy (code 44188)?

No. According to the parenthetical notes that follow code 44188, it is not appropriate to report 44188 in conjunction with 44970. The appendectomy would be considered incidental to the colectomy.

(CPT Assistant, April 2006, page 20)

3. The surgeon performed a laparoscopic assisted vaginal hysterectomy using the daVinci® robotic assistance system, is this reported with the unlisted code?

No. Currently the robotic surgical procedures may be reporting using the existing CPT codes based on the procedure performed and the surgical approach used. In this case, the selection of the CPT code would be 58571 or 58573 based on the site of the uterus.

(CPT Assistant, August 2012, page 13)

Chapter 3

Modifiers

This chapter will focus on the following topics:

• Introduction of CPT and HCPCS modifiers

• List of modifiers approved for Physician Use and Hospital Outpatient Use

• Appropriate use of CPT/HCPCS modifiers

Objectives

After completing this chapter, the student should be able to do the following:

1. Identify the purpose of modifiers

2. State the uses of modifiers for surgical procedures

3. Differentiate between the modifiers identified for Hospital Outpatient Use and the complete list of CPT modifiers

4. Differentiate between modifiers 73 and 74

5. Identify the proper use of modifier 59

6. Research Medicare Transmittals and CPT Assistant pertaining to use of modifiers

7. Given a scenario, append a CPT code with the correct modifier

Suggested Student Activities and Assignments

1. Search CMS website () for Medicare Transmittals pertaining to use of modifiers.

2. Listen to AHIMA Audio Seminars (Use of Modifiers is typically offered every year).

3. Discuss articles from Body of Knowledge

Key Points for Lecture Notes

1. Emphasize the difference between modifiers used for hospital outpatients vs. modifiers for physicians. CPT codebooks have a quick reference on the reverse side of the front cover page.

Mention that the dash between the code and modifier is NOT part of the code assignment but used only for ease in reading the written code assignments.

2. Students should answer the following questions to determine if a modifier should be appended to the CPT code:

• Will a modifier add more information regarding the anatomic site (e.g., LT, T5)?

• Will a modifier help to eliminate the appearance of duplicate billing, or the appearance of unbundling?

3. Although the National Correct Coding Initiative (CCI) is discussed in Chapter 4, faculty should briefly introduce the concept during the discussion on modifiers.

4. CPT has changed their guidance for coding for an incomplete colonoscopy, see the reference to using modifier 53 in the Coding Tip before code 45355.

Chapter 3 Test Bank Items

For each of the following, append CPT/HCPCS modifiers to the following CPT procedure codes.

1. Extracapsular cataract extraction with insertion of lens, OS (CPT code 66984).

66984–LT

2. Arthrocentesis, ring finger of left hand (CPT code 20600).

20600–F3 (Note: Remind students that the F and T modifiers are for fingers and toes, not metatarsals and metacarpals)

3. Closed reduction of fractured phalange, 5th digit, right foot (CPT code 28515)

28515-T9

4. Bilateral maxillary sinusotomies (CPT code 31020).

31020–50

5. Arthroscopic medial meniscectomy and chondroplasty of lateral compartment, left knee (CPT codes 29881 and 29877). Note: CCI edits bundled 29877 with 29881 if they are of the same compartment.

29881–LT, 29877-59–LT (without use of modifier –59, the third party payer may bundle 29877 with code 29881) (Also, note that CMS has created HCPCS code G0289 for reporting, and there must be documentation that the surgeon spent at least 15 minutes in the compartment.)

Assign CPT codes to the following procedures. Append modifiers if applicable.

6. Physician performs a suture repair of entropion of left lower eyelid.

67921–E2

7. Physician performs bilateral vasectomy.

55250 (Note: No modifier is needed, code description states “unilateral or bilateral”)

8. The patient reports that her breasts are too large and as a result, she experiences severe back and shoulder pain. The physician performs a reduction mammoplasty.

19318–50

9. Incision and drainage of carbuncle on left hip.

10060 (No modifier, CPT description does not specific site and the procedure was performed on the skin)

10. Operative Report

Preoperative Diagnosis: Cholecystitis with cholelithiasis

Postoperative Diagnosis: Same

Operative Procedure: Laparoscopic cholecystectomy

Indications: This 77-year-old woman experiences upper abdominal pain and has been diagnosed with cholelithiasis. The risks and benefits of the procedure have been explained in detail.

Technique: With the patient under general anesthesia, the abdomen was prepped and draped in the usual fashion. A small infraumbilical skin incision was made, carried down through the adipose tissue. The fascia was opened in the midline, and the peritoneal cavity under direct vision using open laparoscopic technique. There was adequate insufflation of CO2.

A 10-mm trocar was introduced into the upper abdomen to the right of the midline, two 55-mm trocars in the right upper quadrant area under directed camera vision. Examination noted that there were multiple adhesions in the gallbladder area. At this point, I was notified that the patient’s blood pressure was 150/80 and then dropped to 90/55. The blood pressure was stabilized but the decision was to abort the procedure at this time. All trocars were taken out under direct camera vision. The CO2 was desufflated. Infraumbilical incision was closed using 4-0 Vicryl subcuticular sutures, and Steri-Strips. She will be closely monitored and I will contact her primary care physician to discuss her condition.

Assign codes for physician and hospital outpatient

47562-53 (physician reporting); 47562-74 (hospital outpatient reporting)

CPT Assistant Exercises

Directions: Ask the students to answer these questions after referencing CPT Assistant.

1. Can modifier 59 be appended to an unlisted code such as 29999?

No. It is not appropriate to append any modifier to an unlisted code because modifiers provide the means by which the reporting physician can indicate that a service or procedure has been altered by some specific circumstance, but not changed in its definition or code. Unlisted codes do not describe a specific service; therefore, it is not necessary to utilize modifiers.

(CPT Assistant, September 2005, page 9)

2. If everything listed in code 95922 is not performed, can this code still be reported? Would it have to be modified?

CPT code 95922 requires both a passive title and a Valsalva maneuver be performed. If only one or the other is performed, then modifier ‘52’ Reduced services, should be appended to the code.

(CPT Assistant, June 2003, page 11)

3. A patient is seen for recurrent carpal tunnel syndrome. The physician performs a revision right carpal tunnel release with a local ulnar fat pad rotation flap and reports code 64721, Neuroplasty and/or transposition; median nerve at carpal tunnel. The procedure report states that the ulnar fat pad was rotated on the distally based flap and used to cover the median nerve. Would this procedure be reported separately and, if so, which code should be reported?

Code 64721 is the appropriate code to report the carpel tunnel release; however, there is no specific code for fat pad coverage of the median nerve. If the size of the fat pad and the amount of work involved are significant, then modifier 22, Increase Procedural Services, may be appended to code 64721. Records should reflect the size of the flap and amount of work that is over and above the usual procedure. It is not appropriate to report a flap graft service from the Integumentary System for this technique.

(CPT Assistant, August 2009, page 11)

4. Is CPT code 69610 (tympanic membrane repair) considered to be unilateral or bilateral?

Unilateral. If the procedure is performed bilaterally, modifier ‘50’ Bilateral procedure, should be appended.

(CPT Assistant, March 2003, page 21)

5. A physician states that acoustic reflex test of the left ear was performed (CPT code 92568). Would modifier LT be appropriate?

No. As stated in the CPT guidelines for audiologic function tests, all descriptors refer to testing of both ears. Use modifier 52 if the test is applied to one ear instead of two.

(CPT Assistant, June 2004, page 10)

Chapter 4

Surgery

This chapter will focus on the following topics:

• Surgical package

• Separate procedure

• National Correct Coding Initiative (NCCI)

• Modifier usage

• Surgical coding

Objectives

1. Surgery Overview

a. Describe the organization of the surgical section in CPT

b. List components of a surgical package

c. Distinguish between the CPT definition of surgical package and Medicare definition

d. Define and cite examples of “separate procedure”

e. Define National Correct Coding Initiative (NCCI)

f. Differentiate between modifiers for physician use versus modifiers for hospital outpatient use

g. Given a procedure or operative report, correctly assign CPT codes

2. Integumentary System Subsection

a. Identify guidelines for coding excision of lesions

b. Differentiate between benign and malignant lesions.

c. Distinguish between excision and destruction of lesions

d. Differentiate between simple, intermediate, and complex wound repairs

e. Successfully apply guidelines for coding wound repairs

f. Differentiate between the types of skin grafts

g. Differentiate between excisional and incisional breast biopsies

h. Define key terms: lesion, epidermis, dermis, subcutaneous tissue, benign, malignant

3. Musculoskeletal System Subsection

a. Identify the appropriate use of casting/strapping codes

b. Differentiate between surgical and diagnostic endoscopies

4. Respiratory System Subsection

a. Identify guidelines for coding nasal endoscopy procedures

b. Differentiate between direct and indirect laryngoscopies

c. Define bronchoscopy

d. Describe procedures commonly performed with bronchoscopies

5. Cardiovascular System Subsection

a. Differentiate between injection procedures

b. Define CABG and explain the guidelines for coding assignment

c. Describe the documentation necessary to accurately assign codes for pacemaker procedures

d. Identify the components of coding Interventional Radiology

e. Define an AV fistula and indications for the procedure

f. Differentiate between the coding selections for AV fistulas

g. Differentiate between the venous access devices

6. Digestive System Subsection

a. Define ERCP

b. Define proctosigmoidoscopy, sigmoidoscopy, and colonoscopy

c. Describe the methods for removal of tumors/polyps during a colonoscopy procedure

d. Explain the coding guidelines for incomplete colonoscopy procedures

e. State guidelines for coding biopsies with lesion removals during GI endoscopy

f. Differentiate between the surgical procedures codes for treatment of hemorrhoids

g. Define key terms associated with type, presentation, and/or location of hernias

7. Urinary System Subsection

a. Define the various urodynamic procedures

b. Describe the organization of the urinary system subsection

8. Male Genital System Subsection

a. Differentiate between the removals of lesions described in Male Genital Subsection versus those described in the Integumentary System subsection

9. Laparoscopy/Hysteroscopy Subsection

a. Differentiate between laparoscopic and hysteroscopic procedures

10. Nervous System Subsection

a. Differentiate between the spinal injection procedures

11. Eye and Ocular Adnexa Subsection

a. Differentiate between extracapsular and intracapsular cataract extraction

b. Identify the procedures included in the codes for cataract extractions

12. Auditory System Subsection

a. Differentiate between tympanostomy and myringotomy for insertion of ventilating tubes.

Suggested Student Activities and Assignments

1. For each system, provide a list of applicable medical terms to review with students. For example, in the integumentary system have the students search the following website for a definition of verrucae, condyloma or basal cell carcinoma.

2. Assign students a surgical procedure to research and present to the class.

3. Identify Web references that describe surgical procedures. Note that many of the professional associations have websites that provide health information. For example, the American Academy of Otolaryngology:

4. Identify library references for surgical procedures.

5. Show videos of various surgical procedures. Interactive tutorials can be found on the National Library of Medicine’s website (Medline Plus). In addition, Spine-Health has excellent videos to explain spinal fusion.

Key Points for Lecture Notes

1. Encourage students to carefully read any “Notes” that appear in the CPT book.

2. Explain the most common procedures so the students can identify the purpose and technique utilized.

3. Concentrate on key terms.

4. Remind students to not code what they don’t understand.

5. Supplement lectures with anatomic pictures and/or videos (e.g. Medline Plus)

6. When choosing CPT codes, ask students to explain the difference between each of the codes under consideration

Websites for Faculty and/or Students

It is important that students build a library of resources to research surgical procedures (see Web Resources in Appendix A. Medline Plus provides real surgical videos and interactive tutorials.

Chapter 4 Test Bank Items

Part I: Multiple Choice

1. Which of the following procedures can be identified as destruction of lesions?

a. Shaving of epidermal lesion

b. Removal of skin tags

c. Laser removal of condylomata

d. Paring of hyperkeratotic lesions

2. The patient had a total abdominal hysterectomy with bilateral salpingectomy. The coder selected the following codes: 58150 and 58700. The assignment of these two codes together would be referred to as:

a. Global packing

b. Unbundling

c. Maximizing

d. Optimization

3. Patient had a laparoscopic incisional herniorrhaphy for a recurrent reducible hernia. The repair included insertion of mesh. What is the correct code assignment?

a. 49565

b. 49565, 49568

c. 49656

d. 49560, 49568

4. A physician performs a diagnostic laryngoscopy with use of laryngeal mirror. This type of laryngoscopy is called:

a. Indirect

b. Direct

c. Flexible

d. Microscopic

5. A patient was taken to the endoscopy suite. The endoscope was passed into the esophagus and continued into the stomach and into the duodenal bulb. Based on this documentation, what CPT code would be selected to represent this procedure?

a. 43200

b. 43234

c. 43235

d. 43260

6. What is the correct CPT code assignment for electrosurgical removal of three (3) nevi of the arm (size approximately 2.0 cm, 1.5 cm, 0.5 cm)?

a. 11400, 11402, 11402

b. 11056

c. 11200

d. 17000, 17003, 17003

7. The physician documented the following surgical procedure for treatment of chronic otitis media: “Myringotomy with insertion of ventilating tubes in both ears. Performed under general anesthesia.” What is the correct CPT code assignment for this procedure?

a. 69421–50

b. 69421–LT, 69421–RT

c. 69436–50

d. 69433–LT, 69433–RT

8. What is the correct CPT code assignment for hysteroscopy with lysis of intrauterine adhesions?

a. 58555, 58559

b. 58559

c. 58559, 58740

d. 58555, 58559, 58740

9. The physician documents that she changed the cardiac pacemaker battery. In CPT, the battery is called a(n):

a. Generator

b. Electrode

c. Dual system

d. Cardioverter

10. A patient is seen in the emergency department following an accident. The physician documents that the wound required multiple layers and extensive undermining. According to CPT definitions, this type of repair would be classified as:

a. Simple

b. Intermediate

c. Complex

d. Advancement flap

11. An asymmetric nevi, total excision size of 2.0 cm x 3.0 cm was removed from the patient’s back. Pathology report identifies the specimen as “interdermal nevi.” What is the correct CPT code assignment for this procedure?

a. 11603

b. 11602

c. 11403

d. 11406

12. The physician performs an exploratory laparotomy with bilateral salpingo-oophorectomy. What is the correct CPT code assignment for this procedure?

a. 49000, 58940, 58700

b. 58940, 58720-50

c. 49000, 58720

d. 58720

13. The surgeon performs an open thrombectomy of an AV fistula, without revision of the dialysis graft. What is the correct CPT code assignment for this procedure?

a. 36870

b. 36833

c. 36831

d. 36832

Part II: Practice Exercises

Assign CPT codes to the following procedures. Append modifiers if applicable.

14. With the use of laser, the physician removed two benign lesions on the right side of the back

17000, 17003 Index: Lesion, skin, destruction, benign

15. Puncture aspiration of a cyst of the left breast

19000–LT Index: Breast, cyst, puncture aspiration

16. Shaving of 1.5 cm epidermal lesion, scalp

11307 Index: Lesion, skin, shaving

17. Repair of nail bed, third digit-left hand

11760–F2 Index: Repair, nail bed

18. Closed manipulation of right radial shaft fracture

25505–RT Index: Fracture, radius, shaft

19. Arthrotomy into the left ring finger to remove a piece of metal

26080-F3 Index: Arthrotomy, interphalangeal joint

20. Initial repair of incarcerated ventral hernia with insertion of mesh

49561, 49568 Index: Hernia, repair

21. EGD with removal of a piece of a chicken bone

43247 Index: Endoscopy, gastrointestinal, upper, foreign body

22. Hysteroscopy with D&C and removal of fibroid

58558, 58561 Index: Hysteroscopy, surgical with biopsy & Hysteroscopy, removal leiomyomata

23. Cystoscopy, ureteroscopy with lithotripsy and insertion of indwelling ureteral stent

52332, 52353 Index: Cystourethroscopy, lithotripsy & Cystourethroscopy, insertion indwelling ureteral stent

24. The surgeon replaces the peripherally inserted central venous catheter (PICC) through same access.

36584 Index: Replacement, venous catheter, central

25. The surgeon created a twist drill hole for evacuation of a subdural hematoma.

61108 Index: Drill hole, skull, drain hematoma

Part III: Operative Reports

26. Procedure: Esophagogastroduodenoscopy

This 47-year-old patient has had a history of peptic ulcer disease. He has been experiencing nausea and vomiting. The patient was given Demerol, 50 mg IV and Versed 3 mg IV and the GIF-100 video endoscope was inserted and passed without difficulty. The esophagus was normal. The stomach was diffusely erythematous and there were linear erosions noted in the antrum. Random biopsies were obtained. The pylorus was somewhat spastic. The duodenal bulb and second portion of the duodenum were normal. The patient tolerated the procedure well.

43239 Index: Endoscopy, gastrointestinal, upper, biopsy

27. Operative Report

Preoperative Diagnosis: Desired sterility

Postoperative Diagnosis: Same

Operative Procedure: Bilateral vasectomy

Procedure: The patient was premedicated, brought to the OR in the supine position, prepped with Betadine, draped in sterile fashion. 2% Xylocaine was then injected on the left side. A 1-1/2 cm (excised diameter) compound nevus was removed from the skin of the left lower quadrant followed by simple closure. After picking up the vas with two fingers, the incision was made transversely about 3/4 cm long and deepened through the layer of the scrotum to reach the vas. An Allis clamp was used to pick up the vas which was then identified by the feel and look of the tube and this was then cleaned off of the sheets and layers over the same. The artery to the vas was then coagulated. A 1 cm segment of the vas was then isolated and divided between clamps. The cut end of the vas was then coagulated both sides and 2-0 chromic transfixion stitch was taken on each end. Hemostasis was meticulously achieved and then the sheath of the vas was closed over the superior end leaving the lower end outside the sheath. Closure of the scrotum was then carried out in two layers of 3-0 chromic catgut, one for the other layers of the scrotum, one for the skin. A similar procedure was then carried out on the other side and the procedure was then considered complete. The patient was taken to the recovery room in good condition.

55250 for Vasectomy. Note that the MD removed a nevus from the trunk 11402

28. Operative Report

Preoperative Diagnosis: History of recurrent foreskin infection

Postoperative Diagnosis: Same

Procedure: Circumcision

Indications: The patient has had some evidence of recurrent foreskin infection and his wife has had recurrent infections that her gynecologist has recommended that Mr. K. undergo circumcision. The patient presented at this time to complete that recommendation.

Procedure: The patient was taken to the Operating Room and placed in supine position. General anesthetic was initiated. After good anesthesia was achieved the patient’s penis was prepped and draped in the appropriate fashion. A straight hemostat was used to crush the foreskin on the dorsal aspect first. After it had been placed for a period of time the hemostat was released and the crushed segment was then divided. A similar action was performed on the ventral side. This was done down to the desired site of the circumcision. Then a #3-0 chromic suture was placed on the dorsum ventral side connecting the cut ends of tissue. Curved hemostats were used circumferentially around the penis on the right side to the desired length of circumcision. After the tissue was crushed it was divided and then the excess foreskin was removed. Good hemostasis was achieved using the Bovie and the remaining cut ends of the tissue were reapproximated using interrupted #3-0 chromic suture. Similar action was done on the left side. Remaining cut edges of the tissue were reapproximated using interrupted #3-0 chromic sutures. Vaseline gauze was placed at the suture line followed by dry gauze. The patient tolerated the procedure well. There were no complications. The patient left the Operating Room in stable condition.

Follow up: The patient will follow up in my office in 7 to 10 days. He was given a prescription for Darvocet N 100 mg.

54150-52 Note that MD used device (hemostat) to crush the foreskin. CPT Guidelines state that modifier 52 is reported when the circumcision (code 54150) is performed without dorsal penile or ring block

29. Operative Report

Preoperative Diagnosis: Abnormal uterine bleeding

Postoperative Diagnosis: Same

Procedure: Diagnostic hysteroscopy with D& C

There was an approximately 8-mm polyp of the cervix. The remainder of the endocervix was unremarkable. Uterine cavity was somewhat difficult to visualize but no obvious abnormalities. Minimal tissue on D&C.

Patient was taken to the OR with an IV in place, received general anesthesia and was placed on the operating table in semi-dorsolithotomy position with her legs held by staff. She was then prepped and draped. Pelvic exam was performed. Weighted speculum was placed and single tooth tenaculum placed anteriorly on the cervix. Visualization was good. Diagnostic hysteroscopy was introduced into the endocervix on direct visualization and into the intrauterine cavity. The above findings were noted with no obvious pathology. This was withdrawn and cervix dilated to #8 Hagar. Sharp uterine curette was introduced and the uterine cavity systematically curetted with minimal amount of tissue. Bleeding was negligible and procedure was terminated. Patient tolerated the procedure well and was taken to the recovery room in good condition. Estimated blood loss 15 cc.

58558 Note that the D&C is included; do not assign an additional code, it would be “unbundling”

30. Operative Report

History: 22-year-old female, Gravida IV, Para II, AB I, comes in today because of crampy abdominal pain and passing fetus just on the sidewalk outside of the hospital. Apparently her last menstrual period was April 6 and she had been doing well and this problem just started today.

Preoperative Diagnosis: Incomplete abortion

Postoperative Diagnosis: Same

Operation: Dilatation & Curettage

Procedure: The patient was placed on the operating table in lithotomy position, prepped and draped in the usual manner. Under satisfactory intravenous sedation, the cervix was visualized by means of weighted speculum, grasped in the anterior lip with a sponge forceps. Cord was prolapsed through the cervix and vagina and considerable amount of placental tissue was in the vagina and cervix. This was removed. A sharp curet was used to explore the endometrial cavity, and a minimal amount of curettings was obtained. The patient tolerated the procedure well.

59812 Search in the index under Abortion, incomplete

31. Operative Report

Preoperative Diagnosis: Right carpal tunnel syndrome

Postoperative Diagnosis: Same

Operation: Release of right carpal tunnel

This patient has complained of numbness and tingling in her right hand for several months. After a full evaluation, the diagnosis of right carpal tunnel syndrome was made and surgery recommended.

Operation: The patient was brought to the operating room and placed in supine position and regional anesthesia was administered. The patient was prepped and draped in the usual manner. An incision was made over the ulnar side of the patient's right palm over the 4th metacarpal. Extension was made on to the flexor creases of the palm. This was performed in a transverse fashion, so as to prevent scarring contracture from occurring postoperatively. Dissection was carried sharply through the skin and past the dermis. Then, using hemostat and blunt dissection, the palmar tissues were separated to identify the transverse carpal ligament at its most proximal extent. A small hemostat was placed underneath the transverse carpal ligament, and using sharp dissection with a #15 blade, a complete transverse carpal ligament was released. The wound was copiously irrigated using normal saline solution. The skin was closed using a horizontal mattress #5-0 nylon sutures. Xeroform gauze was placed and then a Kerlix was wrapped around the patient. The patient was placed in a volar splint, which was held in place with an Ace wrap. The patient was returned to the recovery room in stable condition.

64721–RT Index: carpal tunnel syndrome

32. Operative Report

Preoperative Diagnosis: Laceration of nerve and tendon, left 5th digit

Postoperative Diagnosis: Ulnar nerve laceration, no tendon laceration, left 5th digit

Operation: Repair of ulnar nerve

Procedure: The patient was brought into the operating room and prepared and draped in the usual sterile manner. A tourniquet was used and inflated to approximately 250 mm of mercury after exsanguination of the hand. Tendons were noted to be completely intact. The nerve was then isolated in both proximal and distal ends and with the use of an operating microscope 9-0 sutures were placed in the epineurium, six through and through sutures placed. When this was finished the nerve was checked for congruity. It should be stated that the nerve was trimmed and the fascicles were lined up end to end as best as possible. After this, copious irrigation was undertaken and bleeders were cauterized. The skin was then closed with 5-0 nylon and a sterile dressing was applied. The tourniquet was let down and a clam digger splint with a rubber band through the nail was placed to ensure range of motion. The patient was discharged to recovery room without complications and there was approximately 15 cc blood loss. No blood replacement. 400 cc of Ringers lactate was used in the case.

64836–F4 for suture and then add 69990 due to the use of an operating microscope

33. Clinic Record

Procedure: Laryngoscopy

This 45-year-old patient is seen the ENT clinic for a chronic sore throat. The patient's mouth is open wide and the tongue held down with a tongue depressor. The laryngeal mirror is inserted into the back of the mouth just above the uvula. I was able to visualize the epiglottis, larynx, and vocal cords. On the larynx appeared a small lesion. The patient is advised to have the lesion removed in the Outpatient Surgery Department on Tuesday.

31505 Diagnostic indirect laryngoscopy

34. Operative Report

Preoperative Diagnosis: Mass, superior aspect of the left breast

Postoperative Diagnosis: Benign mass, superior aspect of the left breast

Operation: Excision

The patient is a female who has had a lump palpable over the superior aspect of the left breast for the past several months. It has been observed in the office. I had done a needle aspiration and did not get any fluid out. After multiple observations, the patient was very concerned about carcinoma and wanted to have this area excised.

Surgical Technique: The patient was lying down supine. The left breast was scrubbed with Betadine scrub and paint and draped in the classical fashion. The patient has a transverse incision near where we are feeling this lump, which was over about the 11 o’clock position, high up in the superior aspect of the left breast. A transverse incision was made underneath the breast tissue and adipose tissue was completely taken out. Hemostatic was ascertained with electrocoagulation. The wound was closed using interrupted 3-0 Vicryl sutures, the skin was closed with subcuticular running 5-0 Dexon. Benzoin and Steri-Strips and a pressure dressing were applied. All counts were normal.

It was the impression of the pathologist that it represented a benign process in the left breast.

19120–LT Excision of mass

35. Operative Report

Preoperative Diagnosis: Internal derangement left knee

Postoperative Diagnosis: Tear of lateral meniscus

Operative Procedure: Left knee arthroscopy, partial meniscectomy

The arthroscope was inserted through the routine superolateral portal as well as an inferomedial portal for insertion of scope and instruments. The knee joint was then examined in routine manner, the medial meniscus was intact. The lateral meniscus was partially detached and this portion was removed. No other defects were noted. The knee was irrigated well using normal saline. The instruments were removed from the knee. Wound closed with #4-0 nylon and dressed. Estimated blood loss 0. Intravenous fluids 1000 cc. Specimen: meniscus. Complications:

29881–LT Arthroscopy with meniscectomy (lateral)

36. Emergency Department Record

This 3-year-old male was carried into the ED by mother who states, “a fish tank fell over on him” cutting his forehead and cheek. No LOC, PEARTL: patient alert and oriented. Patient has a 3 1/2 cm superficial laceration over the right eye across forehead and 1 1/2 cm. superficial laceration on right cheek. Local anesthesia administered, wound irrigated and sutured with 6-0 Nylon.

12013 Simple wound repairs. Add total wound closures since both from same anatomic description and same classification.

37. Emergency Department Report

This patient was brought to the ER from his work where a foreign body is in the right eye. He was wearing safety glasses but noticed a piece of metal flew in his eye. He reports slight irritation to the right eye but no blurred vision.

PEERLA, Fundi without edema. There was no foreign body on lid eversion. Slit lamp shows a foreign body approximately 2 to 3 o’clock on the edge of the cornea. This foreign body appears metallic. Iris is intact. There are no cells in the anterior chamber. Two drops of Alcaine were used in the right eye. Foreign body was removed without difficulty.

Disposition: Foreign body removed from right eye

65222–RT Removal foreign body with slit lamp

38. Operative Report

Preoperative Diagnosis: History of Colon Polyps

Postoperative Diagnosis: Polyp of Colon

Procedure: Colonoscopy and polypectomy

Indications: The patient is a 46-year-old who had a polyp removed a little over a year ago and presents for a follow up at this time.

Findings: The patient was taken to the Procedure Room and placed in the supine position. The patient was given initially 50mg of Demerol and 3mg of Versed. Next, a rectal exam was performed and the scope was introduced. The prep was poor. The scope could be passed up to an area of about 35 cm and a polyp was found. It was removed with a snare and then brought out with the biopsy forceps through that port. This specimen was sent to the Pathologist for further evaluation. The scope was brought around to the ascending colon. I could not get the scope to traverse any further, but I was able to look down into the cecum. I could not find any gross pathologic changes. The patient received an additional Demerol and Versed during the procedure to a total of 75 of Demerol and 9 of Versed. The scope was then carefully withdrawn and the puddles of fluid were evacuated as the scope was withdrawn. Good hemostasis was found at the site of the polypectomy. The scope was then carefully withdrawn. The patient tolerated the procedure reasonably well. There were no complications. The patient left the Procedure Room in stable condition.

Follow up: The patient will follow up in my office in 7 to 14 days. The patient will be given a prescription for Anusol suppositories.

45385 Colonoscopy with removal of polyp via snare

39. Operative Report

Preoperative Diagnosis: Previous bladder cancer

Postoperative Diagnosis: Same

Procedure: Cystoscopy, multiple bladder biopsies

Anesthesia: General

Procedure: The patient was taken to the Operative Suite, placed in dorsolithotomy position, and sterilely prepped and draped in the usual fashion. Cystoscope was inserted into the urethral and it was normal. There was evidence of previous transurethral resection of prostate (TURP). Upon entering the bladder, there was a tumor that was whitish and had the appearance of leukoplakia. The lateral walls of the bladder were very erythematous and biopsies were taken from the right and left areas. These were sent as specimens. Following this, the areas were then cauterized with Bugbee electrode, catheter was instilled, the irrigant returned was clear, and he was sent to the Recovery Room in satisfactory condition.

52204 Index: Endoscopy, bladder. biopsy

40. Operative Report

Preoperative Diagnosis: Right initial inguinal hernia and umbilical hernia

Postoperative Diagnosis: Same

Procedure: This 78-year-old patient was taken to Surgery, where he was prepped and draped in the normal sterile fashion. Incision was made from 2 cm above the pubic tubercle toward the anterior iliac spine and deepened to the external oblique. The external oblique was opened. The patient’s cord was elevated on a Penrose drain. He had a very large direct inguinal hernia, no indirect hernia. All of the areas were freed up, and a piece of mesh was designed in a keyhole fashion and sutured in place with 2-0 Prolene, avoiding the nerve. Irrigation was performed. The external oblique was closed with 2-0 running chromic. Irrigation was performed again. Scarpa’s fascia was reapproximated using 3-0 chromic, and the skin was closed with staples.

The umbilical hernia was then dissected out after an incision was made beneath the umbilicus. The hernia sac was removed. The fascia was closed with figure-of-eight sutures of 0 Prolene. 2-0 chromic was used to tack down the skin and also reapproximate the subcutaneous area. A running subcuticular of 4-0 Vicryl was placed and Benzoin and Steri-Strips were applied. A dry sterile dressing was applied to each. Betadine was applied to the hernia. The patient was returned to the recovery room in stable condition.

49505–RT Inguinal hernia 49585 Umbilical hernia. Explain to students that the implantation of mesh CPT code (49568) is only used with incisional or ventral repairs

41. Operative Report

Preoperative Diagnosis: Chronic laryngitis with polypoid disease

Postoperative Diagnosis: Same

Procedure: Laryngoscopy with removal of polyps

After adequate premedication, the 60-year-old female patient was taken to the operating room and placed in supine position. The patient was given a general oral endotracheal anesthetic with a small endotracheal tube. The Jako laryngoscope was then inserted. There were noted to be large polyps on both vocal cords, essentially obstructing the glottic airway when the tube was in place. The polyps appeared larger on the right cord. Using the straight-cup forceps, the polyps were removed from the left cord first. The polyps were removed from the right cord up to the anterior commissure. There was very minimal bleeding noted. This opened up the airway extremely well. The patient was extubated and sent to recovery in good condition.

31540 Direct laryngoscopy with excision of tumor of vocal cords

42. Operative Report

Preoperative Diagnosis: Osteomyelitis, fifth metatarsal, left

Postoperative Diagnosis: Same

Procedure: Amputation of toe

The patient was brought to the operating room and placed in supine position. After adequate general anesthesia was obtained, the left foot was scrubbed, prepped, and draped in the usual manner. No tourniquet was utilized. A skin incision was made along the lateral border of the fifth metatarsal and carried down to the subcutaneous tissue in line with the skin incision. Bleeders were clamped and electrocoagulated. Dissection was carried down to the base of the fifth metatarsal where an osteotomy was made at the base. The bone was then delivered from the wound and sent to the pathology department. There was erosion of the head of the fifth metatarsal consistent with osteomyelitis. The toe was amputated and the entire specimen was sent to the pathology department. All of the tissues were débrided. The wound was irrigated and hemostasis assured. The subcutaneous tissue was very loosely reapproximated utilizing 4-0 Vicryl suture. The skin was not closed and was allowed to drain. A sterile dressing was applied to the wound. The patient was then transferred to the recovery room in satisfactory condition.

28810–LT Index: Amputation, toe. Note that the modifier T4 would not be correct because it refers to the digits, not the bones of the foot (metatarsals)

43. Operative Report

Preoperative Diagnosis: Right hydrocele

Postoperative Diagnosis: Right spermatocele

Operation: Right spermatocelectomy

Indications for Procedure: This 54 year old male has a history of right-sided scrotal enlargement. Scrotal ultrasound preoperatively was consistent with right hydrocele.

Operation: The patient was brought to the operative suite, placed in supine position and general anesthesia was administered. His scrotum was shaved. He was then sterilely prepped and draped in the usual manner. A transverse incision across the right hemiscrotum was then made approximately 3.5 cm in length using electrocautery to further dissect this area. The right-sided fluid sac was then exuded from the right hemiscrotum. It seems to be a right spermatocele. Using meticulous care and caution, the spermatocele was divided from the testicle and the vas deferens was identified. There as a moderate degree of difficulty as the spermatocele had separated the epididymis from the patient’s right testicle. So using meticulous care, this was divided free from his spermatocele. The spermatocele was handed off intact to the scrubbed personnel. Hemostasis was achieved. The epididymis was then re-attached to the testicle. The testicle was then replaced into the right hemiscrotum. The wound was closed using a #2-0 locking running chromic stitch and the superficial skin was closed in a horizontal mattress fashion. Patient tolerated the procedure well and was sent to recovery in satisfactory condition.

Pathology Report: spermatocele

54840 Index: spermatocele (good case to explain the difference between the preoperative and postoperative diagnosis)

Chapters 5, 6, 8, 9

Radiology, Pathology and Laboratory, Medicine, Anesthesia

This lesson will focus on key concepts in the following topics:

• Radiology

• Pathology and laboratory

• Medicine

• Anesthesia

Objectives

1. Radiology

a. Apply modifiers associated with radiological procedures (26, TC)

b. State the meaning of the phrase “supervision and interpretation” as it applies to radiological procedures

2. Laboratory

a. Apply physician-billing guidelines for coding laboratory services

b. State the appropriate use of organ- or disease-oriented panels

c. Select pathology level codes associated with pathological examination and diagnosis

3. Medicine

a. Assign codes to report the administration of vaccines or toxoids

b. Apply guidelines to identify injections and infusions

4. Anesthesia

a. Apply “qualifying circumstances” and “physical status” codes to anesthesiology services

b. Given a procedure/service successfully assign CPT code(s) for the above listed services (radiology, pathology/laboratory, medicine, and anesthesia)

Suggested Student Activities and Assignments

1. Research coding references available from specialty professional organizations.

2. Assign the students a specialty area to present to the entire class.

3. Obtain a printout of chargemaster files or superbills and assign the students a quality review exercise.

Key Points for Lecture Notes

1. This lesson covers radiology, pathology and laboratory, medicine, and anesthesia (chapters 5, 6, 8, and 9 in Basic Current Procedural Terminology and HCPCS Coding).

2. Because many of the services covered in this lesson are billed through the use of a chargemaster or super bill, coders do not frequently assign the specific codes. However, the coder should be aware of the pertinent conventions as they apply to each section of the CPT book.

3. After graduation, if a student is employed by a radiologist, anesthesiologist, or pathologist then they will need a more in-depth lesson for these sections. Also, note that many professional organizations (for example, the American Society of Anesthesiologists) provide supplemental materials for CPT coding with their specific specialty.

Chapter 5, 6, 8, and 9 Test Bank Items

1. A patient undergoes a retrograde urethrocystogram. The same physician performs both the injection and the supervision and interpretation. What is the correct CPT code assignment for this physician?

a. 51610, 74450–26

b. 51610, 74450

c. 74450–26

d. 74450

2. A single-view, frontal X-ray of the chest was taken and the radiologist provided only the supervision and interpretation for the procedure. What is the correct CPT code assignment for the radiologist’s services?

a. 71010

b. 71010–TC

c. 71010–26

d. 71015

3. The radiologist provides only the supervision and interpretation of a hysterosalpingography. What is the correct CPT code assignment for the radiologist?

a. 74740

b. 74740-26

c. 58340, 74740

d. 58340, 74740–26

4. A physician draws blood to test for levels of T3 on a non-Medicare patient. The blood is sent to an outside laboratory for analysis. When billing for the physician’s services, which of the following modifiers should be appended to CPT code 84480?

a. 52

b. 59

c. 90

d. 91

5. What is the correct code assignment for bilateral EMG of cranial nerves?

a. 95867

b. 95867–50

c. 95868

d. 95868–50

6. Physician orders part of a Hepatic Function Panel: Serum Albumin, Total Bilirubin, Direct Bilirubin and SGPT, SGOT. What is the correct CPT code assignment?

a. 80076

b. 80076–52

c. 82040, 82247, 82248, 84155, 84460, 84450

d. 82040, 82247, 82248, 84460, 84450

7. The pathologist performed a gross and microscopic examination of a kidney biopsy. What is the correct CPT code assignment?

a. 88300, 88305

b. 88305

c. 88307

d. 88300, 88307

8. A patient with Medicare insurance undergoes a modified radical mastectomy. What would be the correct CPT code assignment for the anesthesiologist’s services?

a. 00474

b. 00406

c. 00404

d. 00400, 00406

9. What is the correct CPT code assignment for IM injection of Leukine?

a. 96401

b. 96372

c. 96374

d. 96369

(Comment: An additional J code would be assigned to identify the specific drug. Students will have to determine the purpose of Leukine (prevent infection in cancer patients) before assigning the correct code.

For each of the following problems, record the correct CPT code(s). Assign modifiers if applicable.

10. A MRI of brain (without contrast material) was performed to rule out the diagnosis of cerebral vascular accident

70551 Index: MRI to Magnetic Resonance Imaging, brain

11. Ultrasound of the gallbladder

76705 Index: Ultrasound, abdomen

12. Chlamydia culture

87110 Index: Chlamydia culture

13. MRI of pelvis, with contrast material

72196 Index: Magnetic Resonance Imaging, pelvis

14. Lipid panel to include: total serum cholesterol, triglycerides and HDL levels

80061 Index: Organ or Disease-Oriented Panel, Lipid Panel

15. Gross and microscopic examination thoracic lymph node biopsy specimen

88305 Index: Pathology, Gross and Micro Exam

16. Pregnancy test performed by urinalysis

81025 Index: Pregnancy test, urinalysis

17. Administration of tetanus toxoid by IM injection for a 34-year-old patient.

90471, 90703 Index: Immunization Administration, one vaccine/toxoid and Vaccines, tetanus toxoid

18. Medical nutrition therapy (initial assessment and intervention); total of 1 hour

97802 x 4 units Index: Nutrition Therapy, Initial Assessment

19. Anesthesia for repair of cleft palate

00172 Index: Anesthesia, cleft palate repair

20. Anesthesia for permanent transvenous pacemaker insertion

00530 Index: Anesthesia, pacemaker insertion

21. A 35-year-old patient undergoes an incisional hernia repair (lower abdomen). Assign the anesthesiology code only.

00832 Index: Anesthesia, Hernia Repair, Abdomen, lower

22. Chiropractic manipulation treatment of three spinal regions

98941 Index: Manipulation, Chiropractic

23. Individual inpatient psychotherapy, 45 minutes

90818 Index: Psychiatric treatment, individual, insight-oriented, hospital

24. Athletic training evaluation

97005 Evaluation and Management, Athletic Training, evaluation

25. Three hour glucose tolerance test (GTT)

82951 Glucose, Tolerance Test

26. Patient screened for tobacco use and received cessation counseling

4004F Patient screened for tobacco use AND received tobacco cessation counseling, if identified as a tobacco user

27. A 62-year-old patient is seen in the outpatient hemodialysis clinic for 3 face-to-face visits in the month of July for treatment of end-stage renal disease. Assign the correct code to reflect services performed for the month of July.

90961 Dialysis, End Stage Renal Disease

28. 30-minute IV infusion of 2 g of Rocephin

96365 Infusion Therapy, intravenous (Rocephin is an antibiotic)

(In addition, J code representing the drug would be assigned)

29. Evaluation of auditory rehabilitation status, 1 1/2 hour visit

92626, 92627, 92627 Rehabilitation, Auditory, status evaluation

30. Vaginal ultrasound

76830 Ultrasound, vaginal

31. Screening for arsenic and mercury

83015 Arsenic

Chapter 7

Evaluation and Management Services

This chapter will focus on the following topics:

• Documentation guidelines

• New versus established patient

• Components of E/M services

Objectives

After completing this chapter, the student should be able to do the following:

1. Describe the contents of the Evaluation and Management (E/M) Services section

2. Differentiate between a new and established patient

3. Identify the seven components included in the descriptors for the levels of E/M services

4. List the three key components for levels of E/M services

5. Describe the documentation requirements for E/M key components

6. State the circumstances when “time” is a key factor in determining E/M level of service

7. Given a scenario, assign the appropriate E/M code

Suggested Student Activities and Assignments

1. Review AMA’s website for current information about documentation guidelines.

2. Research topic on Internet including new proposal for facility E/M guidelines.

3. Invite guest speakers from physician’s office settings.

Key Points for Lecture Notes

1. Introduction

a. This unit is very complex and difficult to understand at first glance. For this reason, the chapter is at the end of the textbook.

b. Evaluation and Management (E/M) services are the most common codes used in the physician’s office. In the hospital setting, E/M codes are used to describe the services for the emergency department visits for the facilities’ cost in providing care. Traditionally, hospital coders have little experience with E/M coding because these services are part of the computerized chargemaster file and the emergency department (ED) physician and/or staff takes the responsibility for deciding the correct code assignment. This has changed with the implementation of the APC system.

2. E/M in the beginning the CPT book.

a. E/M codes were introduced in 1992, but came with no documentation guidelines. Clinical Examples (Appendix C) gave the physicians some guidance, but it was not until 1995 that AMA and HCFA provided official documentation guidelines. Revised guidelines were published in 1997. Currently, physicians can use either the 1995 or 1997 guidelines.

3. E/M services. Ask the students to read the guidelines and definitions for E/M coding and to become familiar with the format and organization. Selection of the appropriate code is based on:

a. Type of service, such as: office visit, consultation, preventive medicine

b. Where service provided, such as: office, hospital, skilled nursing facility

c. Extent: physician document should support the work involved with the case

4. New versus established patient. A glance at the descriptions in the beginning of the E/M section will reveal the categories of new and established patient. The criterion for determining whether a patient is new is based on a 3-year rule. If the patient has not received professional services (face-to-face) by his physician or any other physician of the same specialty (or subspecialty) in the group, then the patient is considered new. If the patient has received services, then the patient is established. Consider the following two scenarios and ask the students to determine if the patient is new or established:

Case #1- Office Visit (1/11/11) Dr. Smith --Patient last seen by this primary care physician (Dr. Smith) on 12/22/09 for strep throat. Patient is now seen for dermatitis of unknown origin. Dr. Smith elects to refer this patient to another physician in his group practice who is a dermatologist (Dr. Jones).

Case #2-Office Visit (1/12/11) Dr. Jones Patient treated for dermatitis.

Answer: On the first visit at Dr. Smith’s office, the patient would be considered established since the patient was treated less than 3 years ago.

Dr. Jones’ visit would be considered new since a different specialist in the group practice saw the patient. If Dr. Jones had been another primary care physician, then the patient would have been considered established even though Dr. Jones had never seen the patient before.

Refer students to the decision tree in CPT for new vs. established patient.

5. Key components. This is where the process gets complicated. The three key components are:

• History

• Examination

• Medical decision making

Point out that the above-mentioned components drive the selection process. Review of the E/M codes reveals such terminology as: problem-focused, expanded problem-focused, detailed, and comprehensive. E/M codes start with an uncomplicated type of visit (i.e., dressing change) to the most comprehensive case (chest pain). The physician’s documentation must support the level chosen. AMA/CMS documentation guidelines state exactly how many of the elements must be documented to qualify for the different levels. It is an attempt to quantify something that is not easily quantified. Another underlying key consideration is medical necessity. If the patient comes to the physician because of an ingrown toenail, a high level of service would not be appropriate. The same would be true of a patient seen with severe abdominal pain. You would expect a high level of E/M code (99214 or 99215) not a low level for treatment of severe abdominal pain.

6. Focus on documentation requirements. The PowerPoint presentation and the Chapter Tables outline the elements necessary to support the level of service.

7. Time. Note in the E/M code descriptions the mention of “time.” Time only becomes a factor when counseling or coordination of care dominates (more than 50 percent) the visit. When time dominates the visit, then “time” becomes the controlling factor for code selection.

Example: An established patient is seen by her family physician to discuss management of medications. Counseling and coordination dominates the visit. The physician documents that the visit took 15 minutes, and 10 minutes of the visit was for counseling. Time becomes the key factor, and code 99213 would be assigned. In other words, documentation of history, examination, and medical decision making does not determine the code selection for this case.

8. E/M and APCs. With implementation of APC methodology, hospitals were required to develop a system for consistent assignment of E/M codes to represent the facilities’ services. Most hospitals have created a point system to systematically access the resources necessary to treat the patient.

9. A consultation initiated by a patient and/or family (not by physician) is NOT assigned a code from the Consultation section. In this case, coders are referred to other CPT sections.

10. New in 2013: In addition to physicians, many E/M codes include “other qualified health care professionals,” in the description of the code.

Chapter 7 Test Bank Items

1. A patient was seen by his family practitioner two years ago. A cardiologist in the same group practice now sees this patient for the first time. For E/M code selection for the cardiologist, the patient would be classified as:

a. New

b. Established

2. Patient is seen by her primary care physician for headaches. The physician performs a physical exam, reviews data, and outlines management options. Which of the following key components is missing from this case?

a. Time

b. History

c. Medical decision making

d. Coordination of care

3. Which of the following is a key factor for selecting a correct code for preventive medicine services?

a. Physician time

b. Coordination of care

c. Age of patient

d. Review of systems

Assign E/M codes to the following scenarios.

4. Office Visit

Date of service: 1/3/12 Last date of treatment: 2/12/09

The patient is seen for a cough and sore throat. The physician performs a problem-focused history, expanded problem-focused examination, and medical decision-making is straightforward. What is the correct E/M code for this service?

99212 Established patient

5. Office Visit

Date of service 11/24/12 Last date of treatment: 7/12/11

The patient is seen for a routine blood pressure check. Nurse documents BP: 135/90. Nurse asks about diet and exercise program. Patient offers no complaints. What is the correct E/M code for this service?

99211 Established patient

6. Office Visit

Date of service: 9/28/12 Last date of treatment: 8/3/08

The patient is seen for a chief complaint of shortness of breath and fatigue. The physician performs a detailed history, comprehensive examination, and medical decision-making is of moderate complexity. What is the correct E/M code for this service?

99203 New patient (History would have to be comprehensive for assignment of 99204)

7. Skilled Nursing Home Visit

Date of service: 1/9/12 Last date of treatment: 12/22/11

Physician performs a detailed interval history, comprehensive examination, and medical decision-making is of moderate complexity. In addition, the physician reviewed the medical record and the recent lab results. What is the correct E/M code for this service?

99309 Subsequent nursing facility care, per day

8. Rolling River Community Village-Patient Visit

Physician sees a new patient in the independent living area of this retirement community. The physician performs a detailed history, problem-focused examination and medical decision-making is of moderate complexity. What is the correct E/M code for this service?

99325 New Patient; Domiciliary, Rest Home, or Custodial Care Services

9. Consultation

Cardiologist asked to render an opinion for a new patient who was admitted to the hospital. The physician performs a comprehensive history and physical examination and the medical decision-making was of moderate complexity. What is the correct E/M code for this service?

99254 Inpatient Consultation

10. Physician Office Record

Physician monitors the management of a patient who it taking long-term warfarin therapy. During this initial 90-day period, the physician monitors the dosage with appropriate testing. What is the correct E/M code for this service?

99363 Anticoagulant Management

Chapter 10

HCPCS Level II

This chapter will focus on the following topics:

• Guidelines for use of HCPCS Level II

• HCPCS Level II modifiers

Objectives

1. Identify the structure of HCPCS Level II codes

2. Successfully apply general guidelines for HCPCS Level II coding assignment

3. Describe the use of HCPCS Level II modifiers

4. Identify the HCPCS Level II modifiers required for hospital outpatient services

Suggested Student Activities and Assignments

1. Review Medicare Bulletins for HCPCS Level II coding guidelines.

2. Review superbills for use of Level II codes.

3. Faculty may elect not to have students purchase the HCPCS Level II code book but instead use an encoder or place a copy on reserve in the library. In addition, the following link provides an electronic version of HCPCS codes:



Key Points for Lecture Notes

1. Introduction. Take time to review the contents of the HCPCS Level II codebook. The Centers for Medicare and Medicaid Services (CMS) revises Level II codes every year. Note that the exercises in this guide are from the 2012 edition of HCPCS.

2. Organization. Level II codes are alphanumeric (letters A–V). Review with the students the following characteristics:

a. D codes are copyrighted by the American Dental Association.

b. C codes are temporary codes for use with outpatient prospective payment only.

c. G codes are also temporary codes that CMS assigns for procedures and services that are being considered for inclusion in CPT.

Example: G0105 Colorectal cancer screening: colonoscopy on individual at high risk

d. Similar to CPT, there is an alphabetic index for locating codes.

e. Level II codes are part of the HCPCS coding system.

f. Because CPT lacks many codes for non-physician procedures and services, CMS created codes to supplement CPT.

3. National modifiers. In addition to the five-digit alphanumeric codes, HCPCS Level II also contains modifiers. The modifiers are two digits and are either alphanumeric or strictly alphabetic.

4. Level III Local Codes. Mention that these codes were eliminated in 2004.

Chapter 10 Test Bank Items

Note that the following answers are from the HCPCS Level II 2003 edition. Unless otherwise noted, assume all services/supplies are for Medicare patients.

1. Face tent for oxygen

A4619 Index: Face, tent, oxygen

2. Injection of 0.5 mg of digoxin

J1160 Index: Digoxin

3. Injection of Ampicillin, 500 mg

J0290 Index: Ampicillin

4. Accu-Check home blood glucose monitor

E0607 Index: Monitor, blood glucose, Accu-Check or AccuCheck, blood glucose meter

5. Cervical collar; foam, unadjustable

L0120 Index: Collar, cervical, nonadjust (foam)

6. Surgical boot for 2-year-old child

L3209 Index: Surgical, boot

7. Two feet of oxygen tubing

A4616 x 2 units (per foot) Index: Oxygen tubing

8. Hearing aid, monaural, behind the ear

V5060 Index: Hearing devices

9. Nebulizer with compressor

E0570 Index: Nebulizer, compressor, with

10. Accessory tray for wheelchair

E0950 Index: Wheelchair, tray

11. Pair of full-length surgical stockings

A4510, A4510 Index: Surgical stocking (code description states “each,” therefore report twice)

12. Screening colonoscopy for patient with history of colon cancer

G0105 Index: Colonoscopy, patient at high risk

13. Speaking valve for status-post tracheostomy patient

L8501 Index: Tracheostomy, speaking valve

14. Screening proctoscopy

S0601 Index: Screening

Chapter 11

Reimbursement in the Ambulatory Setting

This chapter will focus on the following topics:

• Reimbursement in ambulatory settings

• Medicare outpatient code editor

• Quality review

Objectives

1. Explain the prospective payment system utilized for Medicare reimbursement for ambulatory surgery procedures

2. Identify the characteristics of the resource-based relative value scale (RBRVS) for Medicare reimbursement for physicians

3. Identify the characteristics of ambulatory patient classification (APC) groups

4. Describe the purpose of the Medicare outpatient code editor

5. Define unbundling

6. Apply coding skills to detect errors in submitted data

Suggested Student Activities and Assignments

1. Research current information about prospective payment system.

2. Go to the Centers for Medicare and Medicaid Services (CMS) website: . Determine the names and addresses of the Medicare Part A fiscal intermediary and Part B carriers for students’ state. Access the websites for both carriers.

3. Invite guest speakers to discuss the impact of the outpatient prospective payment system.

4. Develop encoding project (using Exercise 11.1) so that students can see the impact of their coding decisions.

5. Discuss articles from Body of Knowledge, such as:

• “Care and Maintenance of Chargemasters,” AHIMA Practice Brief, March 30, 2010

6. Exercise 11.1 provides students the opportunity to code diagnoses using ICD-10-CM.

Key Points for Lecture Notes

1. Introduction. This unit provides an overview of the use of CPT codes in the reimbursement process. In addition, part of the unit is devoted to ongoing quality improvement of coded data.

2. Reimbursement. The purpose of this chapter is to provide an overview of the reimbursement systems that utilize CPT/HCPCS coding systems. The textbook does not provide an in-depth explanation of the ambulatory payment classification (APC) system or the RBRVS for physician services. The text provides definitions of common terms and examples to allow students to recognize the connection between coding and reimbursement. Students should be encouraged to use the CMS website to research current memoranda pertaining to reimbursement and coding.

3. Quality of coded data. Quality of coding is a major concern of health care providers and insurance carriers alike. The exercises provided in the textbook will allow the students to practice their skills of reviewing previously coded data. Ask the students to compare the assigned codes with the guidelines provided in the textbook to determine errors. Remind the students that coded data must be supported by documentation.

4. Unbundling. Unbundling is the practice of coding services separately that should be coded together as a package because all parts are included within one code and, therefore, one price. Unbundling done deliberately to obtain higher reimbursement by using the unbundled codes is a misrepresentation of services and can be considered to be insurance fraud.

5. Typical types of errors

a. Failure to review the entire record for supportive documentation

b. Selection of incorrect code

c. Data entry errors

d. Assigning codes that are not validated by documentation

6. Suggestions for improving the quality of coding

a. Maintain current coding policy and procedure manual

b. Address process for reviewing the health record

c. Require use of coding references

d. Provide ongoing education for coding staff

e. Develop a procedure for clarifying ambiguous documentation

f. Perform ongoing auditing

7. Transition to ICD-10-CM

An optional activity for coding ICD-10-CM for diagnoses has been added to Chapter 11 of Basic CPT. The students can use the electronic files located on the CDC website:

Chapter 11 Test Bank Items

Quality Review of Coded Data. The following are excerpts from HCFA-1500 billing forms containing ICD-9-CM diagnosis codes linked to HCPCS codes. In each problem, verify the CPT code assignment. If the code(s) are incorrect, provide the correct coding assignment. The ICD-9-CM codes are provided for reference only.

1. Patient being treated for spontaneous abortion has a D&C.

| |

|21. Diagnosis or nature of illness or |

|injury |

| |

|1. __634.91____ 3. _____ |

| |

|2. ______ 4. ______ |

| CPT/HCPCS |

| | | | | | |

| | | | |58120 | |

| | | | | | |

58120 is for a nonobstetric D&C, correct code is 59812

2. The physician repaired the 2.0 cm superficial laceration of the forehead and 2.5 cm laceration of the scalp with simple wound closures.

| |

|21. Diagnosis or nature of illness or |

|injury |

| |

|1. _873.42_____ 3. _____ |

| |

|2. _873.0____ 4. ______ |

| CPT/HCPCS |

| | | | | | |

| | | | |12013 | |

| | | | | | |

These two wound repairs cannot be added because they are not from the same anatomic site description. Correct codes: 12011 & 12001

3. Patient with menorrhagia had a laparoscopic lysis of adhesions of fallopian tube and excision of benign tumor of ovary

| |

|21. Diagnosis or nature of illness or |

|injury |

| |

|1. _626.2_ 3. _____ |

| |

|2. ______ 4. ______ |

| CPT/HCPCS |

| | | | | | |

| | | | |58662 | |

| | | | |58660 | |

58660 is a “separate procedure” and should not be assigned with 58662. This is an example of unbundling. Correct code: 58662 only.

4. The surgeon excises a benign breast mass from the left breast for a patient

| |

|21. Diagnosis or nature of illness or |

|injury |

| |

|1. 611.72 3. _____ |

| |

|2. ______ 4. ______ |

| CPT/HCPCS |

| | | | | | |

| | | | |19125–LT | |

| | | | | | |

Code 19125 is for excision identified by preoperative marker (not documented). Correct code 19120–LT

5. Patient has a torn medial meniscus. The physician performs a left medial meniscectomy.

| |

|21. Diagnosis or nature of illness or |

|injury |

| |

|1. _836.0 3. _____ |

| |

|2. ______ 4. ______ |

| CPT/HCPCS |

| | | | | | |

| | | | |29880–LT | |

| | | | | | |

Code 29880 is for meniscectomy both medial and lateral. The correct code assignment would be for medial only, 29881–LT \

6. Physician excises a 2.5-cm lesion (basal cell carcinoma) from the patient’s left arm. The excised margins extended 0.5 cm from around the lesion. A simple repair is used to close the wound.

| |

|21. Diagnosis or nature of illness or |

|injury |

| |

|1. _173.61 3. _____ |

| |

|2. ______ 4. ______ |

| CPT/HCPCS |

| | | | | | |

| | | | |11403–LT | |

| | | | |12001–LT | |

There are several errors with this code assignment. 11403 is excision of benign lesion and this case specifies excision for a malignant lesion. The margins would be included in the dimension of the excised lesion; therefore the size should be reported as 3.5 cm (2.5 + .5 + .5). A simple wound repair is included in the excision of lesion code. Modifier LT is not appropriate when the narrative definition of a code indicates that the procedure applies to different body parts and of the skin. Correct code assignment: 11604

7. Established patient seen in the physician’s office for sore throat and a temperature. The physician performed a problem-focused history, expanded problem-focused examination and medical decision-making was straightforward. The final diagnosis was acute pharyngitis.

| |

|21. Diagnosis or nature of illness or |

|injury |

| |

|1. __462_ 3. _____ |

| |

|2. ______ 4. ______ |

| CPT/HCPCS |

| | | | | | |

| | | | |99213 | |

| | | | | | |

Documentation does not support 99213. Problem-focused history and straightforward decision-making describes code 99212. Correct code assignment: 99212

8. Patient taken to the outpatient surgical site with a diagnosis of chronic hoarseness. Physician performs a flexible bronchoscopy with cell washings and brushings.

| |

|21. Diagnosis or nature of illness or |

|injury |

| |

|1. 784.49_ 3. _____ |

| |

|2. ______ 4. ______ |

| CPT/HCPCS |

| | | | | | |

| | | | |31622 | |

| | | | |31623 | |

Code 31622 is a “separate procedure” and should not be assigned with 31623. This is an example of unbundling. Correct code assignment: 31623

9. Patient with a diagnosis of rectal bleeding has a diagnostic flexible colonoscopy with snare removal of polyp.

| |

|21. Diagnosis or nature of illness or |

|injury |

| |

|1. _569.3_ 3. _____ |

| |

|2. ______ 4. ______ |

| CPT/HCPCS |

| | | | | | |

| | | | |45382 | |

| | | | |45383 | |

There are two errors in this exercise. Code 45382 should not be assigned because documentation does not support control of bleeding. Code 45383 is incorrect for snare removal. Correct code assignment: 45385

10. A patient with Medicare insurance has a diagnosis of ptosis of upper eyelid. In the hospital ambulatory surgical site, the physician performs a blepharoplasty on the left upper eyelid.

| |

|21. Diagnosis or nature of illness or |

|injury |

| |

|1. 374.30_ 3. _____ |

| |

|2. ______ 4. ______ |

| CPT/HCPCS |

| | | | | | |

| | | | |15820–E2 | |

| | | | | | |

There are two errors in this exercise. Code 15820 is for the lower eyelid; code 15822 is for the upper eyelid. Also, modifier –E2 is not correct.

Correct code assignment: 15822–LT (CPT code description identifies “upper eyelid”), so LT would be appropriate; modifier E1 would be redundant.

Answers to Additional Practice Exercises, Appendix C—2013

Case Number Code(s) Index Entries

1. 42825 Tonsils, excision; or Excision, tonsils; or Tonsillectomy

2. 50590-RT Lithotripsy, kidney; or Kidney, lithotripsy

3. 43239 Endoscopy, gastrointestinal, upper, biopsy; or Biopsy, gastrointestinal, upper, endoscopy

4. 23650-RT Dislocation, shoulder

5. 66984-RT Cataract, removal/extraction, extracapsular

6. 31622 Bronchoscopy, exploration

7. 57421 Colposcopy, with biopsy(s) of vagina/cervix

8. 58671 Laparoscopy, oviduct surgery; D&C, uterus

58120 Dilation and Curettage, corpus uteri

58301 Removal, intrauterine device (IUD)

Note: Modifier –51 is for physician office use only.

9. 52601 Prostate, excision, transurethral; or Prostatectomy, transurethral

10. 21356-LT Fracture treatment, zygoma, open treatment

11. 42415 Parotid gland, tumor, excision

12. 29881-LT Arthroscopy, surgical, knee

13. 31541 Laryngoscopy, direct

42960 Hemorrhage, control, oropharynx

Note: Modifier –51 could be used for the physician’s claim.

14. 43246 Endoscopy, gastrointestinal, upper, tube placement

15. 35301-RT Thromboendarterectomy, carotid artery

16. 59320 Cerclage, cervix, vaginal

17. 19307-LT Mastectomy, modified radical; or Breast, excision, mastectomy

Note: Modifier -58 could be used for the physician’s claim.

18. 31526 Laryngoscopy, direct

19. 45378 Colon, endoscopy, exploration

20. 21556 Excision, tumor, neck

21. 45385 Colonoscopy, removal, polyp; or Polyp, removal, endoscopic, colon; or Polyp, colon, removal

22. 47563 Laparoscopy, biliary tract, cholecystectomy

23. 43239 Endoscopy, gastrointestinal, upper, exploration

45331–59 Sigmoidoscopy, biopsy

24. 43235 Endoscopy, gastrointestinal, upper, exploration

43450–59 Dilation, esophagus

25. 12011 Wound, repair (sum of lengths)

26. 19120-RT Excision, breast, cyst

27. 11403 Excision, skin, lesion, benign

12032 Wound repair, intermediate

28. 55700 Prostate, biopsy; or Biopsy, prostate

52000-59 Cystourethroscopy

29. 59812 Abortion, incomplete

30. 45330 Sigmoidoscopy, exploration

31. 36558 Insertion, catheter, venous

32. 49652 Hernia Repair, ventral, laparoscopic; or Repair, hernia, ventral, reducible

33. 19125-RT Breast, excision, lesion, by needle localization

19290-RT Breast, localization, needle wire placement

34. 67800-E2 Chalazion, excision, single

35. 64721-LT Carpal tunnel, decompression

36. 54060 Lesion, penis, surgical excision

37. 62311 Injection, see Epidural injection

38. 38525 Excision, lymph nodes

39. 11643 Lesion, skin, excision, malignant

40. 65205-LT Removal, foreign body, external eye

41. 32608 Thoracoscopy, diagnostic with biopsy

31628 Bronchoscopy, biopsy

42. 23120-RT Mumford Operation, see Claviculectomy, partial

43. 11422 Lesion, skin, excision, benign

44. 28515-T8 Fracture, phalanges, toe, with manipulation

28515-T9

45. 23076-RT Tumor, shoulder, excision (lipoma extended beyond layers of skin) Note that these codes are not in numeric sequence

46. 67901-E1 Eyelid, repair, blepharoptosis, frontalis muscle technique (e.g. banked fascia)

47. 26608-LT Fracture, metacarpal, percutaneous fixation

26608-59-LT

48. 33207 Insertion, pacemaker, heart

49. 68811-RT Nasolacrimal Duct, exploration with anesthesia

50. 12032 Wound, repair, intermediate

51. 52204 Cystourethroscopy, with biopsy. Fulguration was used for hemostasis, an integral part of the procedure.

52. 36831 Thrombectomy, Dialysis Graft, without revision

53. 43235-59 Esophagogastroduodenoscopy

43450 Dilation, esophagus

(In this case, a diagnostic EGD was performed, and after the scope was removed, the physician dilated the esophagus with a Maloney dilator. Since the dilation was performed outside of the scope, then combination code of 43249 would be incorrect. Maloney dilation would be coded 43450. (CPT Assistant, Spring 1994, addresses this subject)

54. 11602 Excision, lesion, malignant (8 mm + 5mm + 5mm = 18 mm or 1.8 cm)

12032 Wound repair, intermediate

55. 31541 Laryngoscopy, Direct (with operating microscope or telescope)

56. 45380 Colonoscopy, proximal to splenic flexure, with biopsy

(January 2004 CPT Assistant provided new advice about use of cold knife biopsy forceps. Removal of small polyps via cold knife is assigned CPT code 45380)

57. 12013 Wound Repair, Simple

12042 Wound Repair, Intermediate

58. 62311 Injection, Spinal cord

59. 52281 Cystourethroscopy, Dilation, Urethra

60. 20103-LT Wound, Exploration, Penetrating, Extremity

(The note before the 20100 codes lists the descriptions that are assigned to this range. The clues in the OP report stated that the stab wound was ENLARGED. Coagulation of small vessels was another.)

61. 64595 Removal, Neurostimulator

62. 26055-F2 Trigger finger repair

63. 58558 Hysteroscopy, surgical with biopsy

64. 64421 Injection, Nerve Anesthetic

65. 60220-RT Thyroidectomy, partial (note that hemithyroidectomy is removing only half of the thyroid- one lobe)

66. 59000 Amniocentesis, diagnostic

Answers to Chapter Review Exercises,

Appendix D

Chapter 1: Introduction to Clinical Coding

Chapter 1 Review

1. The American Medical Association (AMA) updates the CPT codes, and the Centers for Medicare and Medicaid Services (CMS) updates the HCPCS National Codes (Level II).

2. Four

3. ICD-9-CM and CPT/HCPCS

4. a. Diagnosis: ICD-9-CM

Procedure: CPT/HCPCS

b. Diagnosis: ICD-9-CM

Procedure: CPT/HCPCS

c. UB-04 (CMS-1450)

5. ICD-9-CM

6. Procedure code 11440 is linked with diagnosis code #1 (216.3). Procedure code 82951 is linked with diagnosis code #3 (790.29).

Note: Depending on the carrier, you may link more than one reference number in block 24E, whereas some payers require just one. When reporting more than one code on a CMS-1500 claim, enter the code with the highest fee in line 1 of block 24 and the remaining in descending order of charges.

Chapter 2: Application of the CPT System

Exercise 2.1 Organization of CPT

 1. Surgery

 2. Evaluation and Management

 3. Pathology and Laboratory

 4. Category II codes

 5. Medicine

 6. Radiology

 7. Anesthesia

 8. Category III codes

 9. Maternity Care and Delivery

10. StandbyServices

11. Radiation Oncology

12. Surgical Pathology

Exercise 2.2 CPT Conventions

 1. Repair blood vessel, direct; hand, finger

 2. Two

 3. Revised descriptor

 4. Esophagoscopy, rigid or flexible, with insertion of plastic tube or stent

 5. New descriptor

 6. No

 7. No

 8. 13152, 13153

 9. 40814

10. 31578, 31576

Exercise 2.3 Use of the Alphabetical Index

 1. 23400 Green operation, see scapulopexy

 2. 10021 Fine Needle Aspiration

 3. 35556 Graft, see Bypass graft, Femoral Artery or Popliteal Artery

 4. 49555 Hernia repair, femoral, recurrent

 5. 26991 Incision and drainage, bursa, hip

 6. 01680 Anesthesia, shoulder

 7. 31525 Laryngoscopy, direct

 8. 72149 MRI see magnetic resonance imaging, spine, lumbar

 9. 11055 Paring, skin lesion, benign hyperkeratotic, single lesion

10. 86701 HIV-1, antibody

11. 41105 Biopsy, tongue

12. 43239 Esophagogastroduodenoscopies, see Endoscopy, gastrointestinal, upper, biopsy

13. 29881 Arthroscopy, surgical, knee

14. 95827 EEG, see Electroencephalography, sleep

15. 65220 Removal, foreign body, cornea, without slit lamp

16. 58356 Endometrial Ablation (this exercise is difficult find in the alphabetic index, but a good lesson on how locating correct entries in the index is not a perfect science). Endometrial ablation directs coder to 58363, which is endometrial ablation (thermal). Alert students to look at the code below 58363—58356.

17. 4008F Beta-blocker therapy, see Performance Measures, Therapeutic, preventive or other interventions, Beta-Blocker therapy

18. 0184T Excision, Tumor, Rectum, Transanal Endoscopic

19. 77082 DXA, see Dual X-ray Absorptiometry, vertebral fracture

20. 25112 Ganglion, Cyst, Wrist Excision

Exercise 2.4 CPT Coding Process

 1. Colonoscopy and polypectomy

 2. 45384–45385

 3. How was the polyp removed (hot biopsy forceps, snare, and so on)?

 4. “Was removed with hot biopsy forceps and retrieved”

 5. 45384

Exercise 2.5 CPT Coding Process

 1. Excision

 2. Can be located under Excision, lesion, skin; or Lesion, skin, excision

Selections: Benign 11400–11471

Malignant 11600–11646

 3. Pathologic diagnosis indicates that the lesion was malignant (11600–11646).

 4. Documentation is needed to code malignant lesion, size of lesion + margins (or size of excision)

(2.0 cm + 0.5 cm + 0.5 cm = 3.0 cm excision site) and site (arm).

 5. 11603

Exercise 2.6 CPT Coding Process

 1. Hernia repair

 2. Index entries: Hernia repair, umbilicus; Repair, hernia, umbilical. Codes to review:

49580–49587

 3. Age of patient; incarcerated or strangulated hernia

 4. Review of documentation (38-year-old patient and no documentation for incarcerated/strangulated hernia). Using the abstracted documentation and process of elimination, the correct code would be 49585.

 5. Wound closure would be an integral part of the procedure and would not be assigned a CPT code.

Exercise 2.7 Coding References

 1. 45380

 2. CPT Assistant, January 1996, page 7, instructs the coder to assign 45385. CPT Assistant, January 2004, states that if a small polyp is removed via cold knife biopsy, the appropriate code is 45380. This is a good example of the need to research the most current coding advice.

Exercise 2.8 Coding References

 1. When a biopsy of a lesion is obtained and the remaining portion of the same lesion is then excised/fulgurated, only the code for the excision/fulguration should be used. When the biopsy is taken from a different lesion than the one excised, the biopsy code and an additional code for the removal of the separate lesion are reported. It would be appropriate to append modifier 59 to the code reported for the biopsy procedure.

 2. Reference: CPT Assistant, October 2004, Skin Biopsy Coding Guidelines.

Exercise 2.9 Coding References

1. No. Code 36479 would be reported only once. The code descriptor for code 36479 states, “second and subsequent veins treated in a single extremity, each through separate access sites,” indicating that the second, third, fourth, etc. vein(s) are represented in code 36479. No additional reporting occurs after the second vein is treated.

2. Reference: CPT Assistant, July 2012, pages 12-13 (Frequently Asked Questions)

Exercise 2.10 Chapter 2 Review

 1. Bullet

 2. 11100 and 11101

 3. Category III

 4. 42320

 5. 40843

 6. Removal of less than 80% of vulvar area, and there was removal of skin and deep subcutaneous tissue. (See the note before CPT code 56405.)

 7. Method of removal

 8. No. Versed is a type of conscious sedation. The symbol appears before code 43235; therefore, administration/monitoring of conscious sedation would be inherent in the procedure.

 9. 76857, based on the definitions provided in the note before code 76830

10. 11730, 11732, 11732

Chapter 3: Modifiers

Exercise 3.1 Chapter 3 Review

 1. 55

 2. 77

 3. 22

 4. 62

 5. 53

 6. 64721–50 Carpal tunnel syndrome, decompression

 7. 11730–FA Avulsion, nails

 8. 11043–73 Debridement, muscle

 9. 64611-52 Chemodenervation, salivary glands

10. 19102–LT Biopsy, breast

11. 28485–RT Fracture, metatarsal, open treatment

Note: –T9 is not applicable in this case because the procedure refers to the bones of the foot, not the toes.

Chapter 4: Surgery

Exercise 4.1 Integumentary System- Debridement

1. 11042 Debridement, skin, subcutaneous tissue

2. 11010 Debridement, skin, with open fracture

3. 11005 Debridement, skin, infected

11008 Removal, mesh, abdominal wall

Exercise 4.2 Integumentary System - Lesions

 1. 11421 Excision, skin, lesion, benign; or

11402 Lesion, skin, excision, benign

 2. 17110 Lesion, skin, destruction, benign

 3. 11642 Excision, skin, lesion, malignant; or Lesion, skin, excision, malignant

 4. 11200 Skin, tags, removal

 5. 11403 Lesion, skin, excision, benign

 6. 11644 Lesion, skin, excision, malignant

 7. 11641 Lesion, skin, excision, malignant

11640

Exercise 4.3 Integumentary System

 1. 11442 Excision, skin, lesion, benign; or Lesion, skin, excision, benign

Exercise 4.4 Integumentary System

 1. 11308 Lesion, skin, shaving

Exercise 4.5 Integumentary System – Wound Repairs

 1. 12002 Wound, repair; or Repair, wound, simple

 2. 12032 Wound, repair; or Repair, wound, intermediate

 3. 12041 Wound, repair; or Repair, wound, intermediate

12002 Wound, repair; or Repair, wound, simple (sum of repairs)

Exercise 4.6 Integumentary System

 1. 12001 Wound, repair; or Repair, wound, simple

Exercise 4.7 Integumentary System

 1. 12032 Wound, repair; or Repair wound (simple and intermediate)

12005

Exercise 4.8 Integumentary System

 1. 13121 Wound, repair (complex)

13122

Exercise 4.9 Integumentary System

1. 11603 Lesion, skin, excision, malignant

12031 Wound, repair (intermediate)

Exercise 4.10 Integumentary System – Skin Grafts

 1. 14021 Skin Graft and Flap, tissue transfer

 2. 15150 Skin Graft and Flap, tissue-cultured

15151

 3. 15220 Skin Graft and Flap, free skin graft, full thickness

Exercise 4.11 Integumentary System

 1. 15120 Autograft, skin, epidermal

11646 Excision, skin, lesion, malignant; or Lesion, skin, excision, malignant

Exercise 4.12 Integumentary System

 1. 15100 Skin, grafts, free

Exercise 4.13 Integumentary System

 1. 19000–LT Breast, cyst, puncture aspiration

Exercise 4.14 Integumentary System

 1. 19125–LT Excision, breast, cyst; or Breast, excision, lesion by needle localization

Note: The hospital also would assign 19290 for placement of the wire. The surgeon would not assign this code because the radiologist performed the procedure. In addition, a radiology code would be submitted for both the facility and the radiologist. Radiology is introduced in chapter 5.

Exercise 4.15 Integumentary System

 1. 19120–RT Breast, excision, cyst; or Excision, breast, cyst

Note: The entire nodule was excised, not just a piece of tissue, which is implied with the term biopsy.

Exercise 4.16 Integumentary System Review Review

 1. 16020 Burns, dressings

 2. 14020 Skin Graft and Flap, tissue transfer

Note: Excision of lesion is included and should not be assigned an additional code.

 3. 12032 Wound, repair, intermediate

Note: Anatomic modifiers (LT, RT) are not appropriate.

 4. 19102–LT Breast, biopsy

19295–LT Breast, metallic localization clip placement

 5. 11606 Lesion, skin, excision, malignant

 6. 11770 Pilonidal cyst, excision (Single-layer closure indicates “simple.”)

 7. 11750–TA Nails, removal

 8. 15781 Dermabrasion

 9. 17273 Skin, destruction, malignant lesion

10. 11043 Debridement, skin, subcutaneous tissue

Exercise 4.17 Musculoskeletal System - Fractures

 1. 25545–LT Fracture, ulna, shaft, open treatment

 2. 21800 Fracture, rib, closed treatment

21800

 3. 23605–RT Fracture, humerus, closed with manipulation

 4. 27532–RT Fracture, tibia, closed treatment (initial cast application included with treatment code)

 5. 27562–RT Dislocation, patella, closed treatment

 6. 27514–LT Fracture, femur, distal

 7. 24516–RT Fracture, humerus, shaft

Exercise 4.18 Musculoskeletal System

 1. 29515–RT Splint, leg, short

Note: E/M code 99283–25 is applicable in this case.

Exercise 4.19 Musculoskeletal System

 1. 27788–LT Fracture, fibula, closed treatment; or Fracture, fibula, with manipulation

Exercise 4.20 Musculoskeletal System - Arthroscopy

 1. 29875–LT Arthroscopy, surgical, knee

 2. 29827–RT Arthroscopy, surgical, shoulder

 3. 29846–LT Arthroscopy, surgical, wrist

 4. 29837–LT Arthroscopy, surgical, elbow

Exercise 4.21 Musculoskeletal System

 1. 29882–RT Arthroscopy, surgical, knee

Exercise 4.22 Musculoskeletal System Review

 1. 28475–LT Fracture, metatarsal, closed

28475–LT

 2. 28285–T1 Hammertoe repair

 3. 27766–RT Fracture, ankle, medial

 4. 20101 Wound, exploration, penetrating, chest

 5. 23030 Hematoma, shoulder, drainage

 6. 24343–RT Ligament, repair, elbow

 7. 28740–LT Arthrodesis, tarsal joint

 8. 29844–RT Arthroscopy, surgical, wrist

 9. 28108–T2 Excision, cyst, phalanges, toe

10. 27372–RT Removal, foreign body, knee joint

Exercise 4.23 Respiratory System - Endoscopy

 1. 31255–50 Ethmoidectomy, endoscopic

 2. 31238–LT Endoscopy, nose, surgical; or Nose, endoscopy, surgical

 3. 31233–50 Sinusoscopy, sinus, maxillary

Exercise 4.24 Respiratory System

 1. 30520 Septoplasty

31267–50 Endoscopy, nose, surgical

30140–50 Turbinate, excision

Exercise 4.25 Respiratory System - Laryngoscopy

 1. 31540 Laryngoscopy, direct

 2. 31577 Laryngoscopy, fiberoptic

 3. 31510 Laryngoscopy, indirect

 4. 31541 Laryngoscopy, direct

Exercise 4.26 Respiratory System

 1. 31536 Laryngoscopy, direct

Exercise 4.27 Respiratory System - Bronchoscopy

 1. 31628 Bronchoscopy, biopsy

 2. 31641 Bronchoscopy, removal, tumor

 3. 31625 Bronchoscopy, biopsy

31623 Bronchoscopy, brushing

Exercise 4.28 Respiratory System

 1. 31623 Bronchoscopy, brushing

Exercise 4.29 Respiratory System Review

 1. 32666 Thoracoscopy, surgical with wedge resection of lung

 2. 30110 Polyp, nose, excision, simple

 3. 31625 Bronchoscopy, biopsy

 4. 30300 Removal, foreign body, nose

 5. 31237 Endoscopy, nose, surgical

 6. 30903–50 Epistaxis

 7. 31540 Laryngoscopy, direct

 8. 32556 Puncture, pleural cavity, drainage

 9. 31576 Laryngoscopy, fiberoptic

10. 30130 Excision, turbinate

Exercise 4.30 Cardiovascular System

 1. 33207 Pacemaker, heart, insertion; or Insertion, pacemaker, heart

Exercise 4.31 Cardiovascular System

 1. 36590 Removal, venous access device

Exercise 4.32 Cardiovascular System

 1. 36556 Insertion, venous access device, central

Exercise 4.33 Cardiovascular System

Review

 1. 33222 Pacemaker, heart, revise pocket, chest

 2. 36833 Arteriovenous Fistula, Revision, with thrombectomy

 3. 33824 Ductus Arteriosus, repair

 4. 36215 Catheterization, brachiocephalic artery

 5. 36582 Venous Access Device, replacement

 6. 36870 Arteriovenous Fistula, thrombectomy, graft; or Thrombectomy, arteriovenous fistula, graft

 7. 37224 Angioplasty, Femoral artery, intraoperative

 8. 33464 Valvuloplasty, tricuspid valve

 9. 33217 Insertion, pacing cardio-defibrillator, leads

10. 37722–50 Vein, stripping, saphenous

Exercise 4.34 Digestive System - Endoscopy

 1. 43202 Endoscopy, esophagus, biopsy

43217 Endoscopy, esophagus, removal, polyp

Note: Modifier 59 would apply.

 2. 45305 Proctosigmoidoscopy, biopsy

 3. 43258 Endoscopy, gastrointestinal, upper, destruction of lesion

 4. 45380 Endoscopy, colon, biopsy

 5. 43264 Bile Duct, endoscopy, removal, calculi

Exercise 4.35 Digestive System

 1. 43247 Endoscopy, gastrointestinal, upper, foreign body

Exercise 4.36 Digestive System

 1. 45384 Colonoscopy, removal, polyp; or Endoscopy, colon, removal, polyp

Exercise 4.37 Digestive System

 1. 45330 Sigmoidoscopy, exploration

Exercise 4.38 Digestive System – Hernia Repairs

 1. 49500 Hernia, repair, inguinal

 2. 49561–RT Hernia, repair, incisional, incarcerated

49568 Implantation, mesh

 3. 49651 Laparoscopy, hernia repair, inguinal, recurrent

 4. 49521 Hernia, repair, inguinal, incarcerated

 5. 49585 Hernia, repair, umbilicus, reducible

 6. 49656 Hernia, repair, incisional, laparoscopic

Exercise 4.39 Digestive System

 1. 49505–LT Hernia, repair, inguinal

Note: Mesh code is only coded with incisional and ventral hernia repairs.

Exercise 4.40 Digestive System Review

 1. 46930 Hemorrhoids, destruction

 2. 43644 Laparoscopy, gastric restrictive procedures

 3. 43245 Endoscopy, gastrointestinal, upper, dilation

 4. 42200 Palatoplasty

 5. 45383 Colonoscopy, destruction, lesion

 6. 47564 Laparoscopy,cholecystectomy

 7. 49521 Hernia Repair, inguinal, recurrent, incarcerated

 8. 49322 Laparoscopy, aspiration

 9. 46610 Anoscopy, removal, polyp

10. 42809 Removal, foreign body, pharynx

Exercise 4.41 Urinary System - Cystoscopy

 1. 52332 Cystourethroscopy, insertion, indwelling ureteral stent

 2. 52234 Cystourethroscopy, with fulguration, tumor

 3. 52353 Cystourethroscopy, lithotripsy

 4. 52282 Cystourethroscopy, insertion, urethral stent

Exercise 4.42 Urinary System

 1. 52352 Cystourethroscopy, removal, calculus

52332–51–RT Insertion, stent, ureteral

Note: Because the stent was inserted at the conclusion of the procedure, one can presume it is an indwelling ureteral stent.

Exercise 4.43 Urinary System

 1. 52234 Cystourethroscopy, with fulguration, tumor

Exercise 4.44 Urinary System Review

 1. 51992 Sling Operation, stress incontinence

 2. 53260 Excision, polyp, urethra

 3. 52235 Cystourethroscopy, with fulguration, tumor

 4. 51525 Cystotomy, excision, bladder diverticulum

 5. 50200 Kidney, biopsy

 6. 52351 Cystourethroscopy

 7. 52290 Cystourethroscopy, with ureteral meatotomy (bilateral modifier not appropriate because code description specifies “unilateral” or “bilateral”)

 8. 51785 Electromyography, sphincter muscles, urethral, needle

 9. 51701 Insertion, catheter, urethral

10. 50920 Fistula, ureter

Exercise 4.45 Male Genital System

 1. 54057 Lesion, penis, destruction, laser surgery

 2. 10060 Incision and drainage, abscess, skin

 3. 55845 Prostatectomy, retropubic, radical

 4. 54861 Epididymectomy, bilateral

 5. 54150 Circumcision, surgical excision, newborn

Exercise 4.46 Male Genital System

 1. 55875 Prostate, brachytherapy, needle insertion

Exercise 4.47 Male Genital System

 1. 54520–LT Orchiectomy, simple

Exercise 4.48 Male Genital System Review

 1. 54840 Spermatocele, excision

 2. 54415 Penile Prosthesis, removal, inflatable

 3. 54690 Orchiectomy, laparoscopic

 4. 54322 Hypospadias, one stage, meatal advancement

 5. 54865 Epididymis, exploration, biopsy

 6. 55100 Scrotum, abscess, incision and drainage

 7. 54240 Penis, plethysmography

 8. 54060 Penis, lesion, destruction, surgical excision

 9. 54640–50 Orchiopexy, inguinal approach

10. 55866 Laparoscopy, prostatectomy

Exercise 4.49 Female Genital System

 1. 58670 Laparoscopy, oviduct surgery

 2. 58120 Dilation and curettage, corpus uteri

 3. 59812 Abortion, incomplete

 4. 49322 Laparoscopy, aspiration

 5. 58545 Laparoscopy, removal, leiomyomata

 6. 58262 Hysterectomy, vaginal, removal tubes/ovaries

Exercise 4.50 Female Genital System

 1. 57461 LEEP Procedure

Exercise 4.51 Female Genital System

 1. 58558 Hysteroscopy, surgical with biopsy

Exercise 4.52 Female Genital System Review

 1. 57455 Colposcopy, biopsy (range of codes)

 2. 56620 Vulvectomy, simple, partial (use definitions at beginning of section)

 3. 58662 Laparoscopy, destruction, lesion

 4. 58150 Hysterectomy, abdominal, total

 5. 57023 Incision and Drainage, hematoma, vagina

 6. 56605 Biopsy, vulva (external genitalia)

57105 Biopsy, vagina

 7. 58554 Hysterectomy, vaginal

 8. 58561 Hysteroscopy, removal, leiomyomata

 9. 58290 Hysterectomy, vaginal

10. 58356 Ablation, endometrium, ultrasound guidance

Exercise 4.53 Endocrine System Review

 1. 60280 Thyroglossal Duct, cyst, excision

 2. 60650 Laparoscopy, adrenal gland, excision

Note: There are several misleading index entries for this example. Adrenalectomy, laparoscopic, leads to 50545. Same is true for Laparoscopy, adrenalectomy.

 3. 60260–50 Thyroid Gland, excision, total, removal of all thyroid tissue

 4. 60300 Thyroid gland, cyst, aspiration

 5. 60500 Parathyroid Gland, excision

Exercise 4.54 Nervous System

 1. 64445 Injection, nerve, anesthetic

 2. 64702–F8 Neuroplasty, digital nerve

69990 Operating microscope

 3. 62281 Epidural, injection

 4. 64831–F5 Neurorrhaphy

64832–F8

Exercise 4.55 Nervous System

 1. 63075 Discectomy

Exercise 4.56 Nervous System Review

1. 62220 Creation, shunt, ventriculo

2. 64712 Neuroplasty, peripheral nerve

3. 64782 Excision, neuroma

4. 64840 Suture, nerve

5. 63272 Laminectomy

6. 62311 Epidural, injection

7. 63688 Neurostimulators, removal, pulse generator

8. 63706 Myelomeningocele, repair

9. 64408 Nerves, injection, anesthetic

10. 61312 Craniotomy, surgery

Exercise 4.57 Eye and Ocular Adnexa

 1. 66984–LT Phacoemulsification, removal, extracapsular cataract

Exercise 4.58 Eye and Ocular Adnexa Review

 1. 65222 Removal, foreign body, cornea with slit lamp

 2. 67810–E1 Biopsy, eyelid

 3. 68110 Lesion, conjunctiva, excision

 4. 67800–E3 Chalazion, excision, single

 5. 67311 Strabismus, repair, one horizontal muscle

 6. 67700–E3 Eyelid, abscess, incision and drainage

 7. 67906–E1 Blepharoptosis, repair, superior rectus technique with fascial sling

 8. 67961 Eyelid, repair, excisional

 9. 67914 Ectropion, repair, suture

10. 67413–RT Orbitotomy, with removal of foreign body

Exercise 4.59 Auditory System

 1. 69436–50 Tympanostomy (–50 for bilateral)

Exercise 4.60 Auditory System Review

 1. 69910 Labyrinthectomy with mastoidectomy

 2. 69205–LT Removal, foreign body, auditory canal, external, with anesthesia

 3. 69636 Tympanoplasty, with mastoidotomy, with ossicular chain reconstruction

 4. 69005–LT Abscess, ear, external, complicated (would expect to see documentation to explain why the procedure took an extensive amount of time)

 5. 69401 Eustachian Tube, inflation, without catheterization

Exercise 4.61 Chapter 4 Review: Coding for Facility

 1. 42305 Incision and drainage, abscess, parotid gland

 2. 69436–50 Tympanostomy

 3. 57455 Colposcopy, biopsy

 4. 52234 Cystourethroscopy, with fulguration, tumor

 5. 19101–LT Biopsy, breast

 6. 12051 Repair, wound, intermediate

 7. 26750–F5 Fracture, phalanges, closed treatment, distal

 8. 31536 Laryngoscopy, direct

 9. 36569 Catheter, venous, central line

10. 45333 Sigmoidoscopy, removal, polyp

11. 49507–50 Hernia repair, inguinal, incarcerated

12. 29834–LT Arthroscopy, surgical, elbow

13. 26010–FA Finger, abscess, incision and drainage

26010–F1

14. 63266 Laminectomy

15. 69666 Fistula, oval window

16. 28261 Capsulotomy, foot

17. 54060 Lesion, penis, surgical excision

18. 11305 Lesion, skin, shaving

19. 59000 Amniocentesis

20. 58558 Polypectomy, uterus

Exercise 4.62 Chapter 4 Review: Coding for Physician Services

 1. 61520–62 (physician #1)

Cerebellopontine angle tumor, see brain, tumor, excision

61520–62 (physician # 2)

 2. 59840 Abortion, induced by dilation & curettage

 3. 29821–RT Arthroscopy, surgical, shoulder

 4. 33208–54 Pacemaker, heart, insertion (modifier for surgical care only)

 5. 43268 ERCP, see Pancreatic duct, endoscopy, tube placement

 6. 58661–22 Laparoscopy, removal, fallopian tube

 7. 68811–50 Nasolacrimal duct, exploration, with anesthesia

Note: Bilateral modifier applies because

the code describes one duct.

 8. 31267–50 Endoscopy, nose, surgical

 9. 44151 Colectomy, total, open, with ileostomy

10. 27786–79 Fracture , fibula, closed treatment

11. 15783 Dermabrasion

Chapter 5: Radiology

Exercise 5.1 Diagnostic Radiology

 1. 74182 MRI, see Magnetic resonance imaging, abdomen

 2. 72131 CT scan without contrast, spine, lumbar

 3. 72170 X-ray, pelvis

 4. 74241 X-ray, gastrointestinal

 5. 74430 Cystography

6. 74261 CT scan, without contrast, colon, colonography

7. 74177 CT scan, with contrast, abdomen (Note: The note under code 74170 directs the coder to assign a code for a combined CT of abdomen and pelvis)

Exercise 5.2 Diagnostic Ultrasound

 1. 76831 Hysterosonography, see Ultrasound; Sonohysterography

 2. 76815 Ultrasound, pregnant uterus

 3. 76645 Ultrasound, breasts

4. 76800 Ultrasound, spine

5. 76770 Ultrasound, kidney

Exercise 5.3 Chapter 5 Review

 1. 73090–26 X-ray, arm, lower

Note: Modifier 26 is reported to identify the professional component of the procedure which includes supervising the procedure, reading and interpreting the results, and documenting the interpretation in a report.

 2. 72142–26 MRI, see magnetic resonance imaging, spine, cervical

 3. 74430 Cystography

Note: Modifier 26 was not appended to the code because the description includes supervision and interpretation.

 4. 78451 Myocardial, Perfusion imaging

 5. 77057 Mammography, screening

 6. 74178 CT Scan, without and with Contrast, Abdomen

 7. 70460–TC CT scan, with contrast, head

Note: In this case, the radiology facility would report modifier TC to identify the technical component of the procedure, which includes performance of the actual procedure and expenses for supplies and equipment.

 8. 76817 Ultrasound, pregnant uterus

 9. 76942 Ultrasound, guidance, needle biopsy

10. 78306 Bone, nuclear medicine, imaging

11. 74270 Barium enema

12. 78205 SPECT, liver

13. 77404 Radiation therapy, treatment delivery, single area

14. 78707 Scan, see Specific site, nuclear medicine, Kidney, Nuclear medicine, imaging

15. 71020 X-ray, chest

16. 77761 Radioelement, application

17. 72240 Myelography, spine, cervical

18. 76775 Ultrasound, retroperitoneal

19. 73540 X-ray, hip

20. 78262 Reflux study

21. 76775 Ultrasound, kidney (Code 76775, limited, is for a single organ.) (See CPT Assistant, May 1999.)

22. 73500–LT X-ray, hip

72170 X-ray, pelvis (1 to 2 views)

23. 71020 X-ray, chest

24. 74247 X-ray, gastrointestinal

25. 70450 CT scan, without contrast, head

Chapter 6: Pathology and Laboratory Services

Exercise 6.1 Chapter 6 Review

 1. 88305 Pathology, gross and micro exam

 2. 83880 Natriuretic peptide

 3. 80402 ACTH, see adrenocorticotropic hormone, stimulation panel

 4. 81003 Urinalysis, automated

 5. 85055 Platelet assay

 6. 88309 Pathology, surgical, gross and micro exam

 7. 86485 Candida skin test

 8. 86689 Antibody, HTLV-1

 9. 80061 Organ- or disease-oriented panel, lipid panel

80051 Organ- or disease-oriented panel, electrolyte

10. 80198 Drug assay, theophylline

11. 82803 Blood gases—pO2, pCO2

12. 86706 Hepatitis antibody, B surface

13. 85730 Thromboplastin, partial time

85610 Prothrombin time

14. 88027 Autopsy, gross and micro examination

15. 88331 Pathology, surgical, consultation, intraoperative

Chapter 7: Evaluation and Management Services

Exercise 7.1 Evaluation and Management (History)

1. HPI is brief (location, quality, and duration). Review of system(s) is problem specific. No PFSH documented. The history component of this visit would be expanded problem focused (brief HPI, problem-specific ROS, and expanded problem-focused PFSH including past, family, and social history). The history component is equal to the lowest category documented.

2. HPI is brief (location, severity, and duration). Review of system(s) is extended (two to nine systems), and PFSH is pertinent (medications). The history is determined by the lowest level from all three categories; therefore, the history level would be expanded problem focused.

3. HPI is extended (location, severity, duration, and context). Review of systems is extended (two to nine systems). The PFSH is complete (two history areas documented). The history level is detailed; all three categories met this level.

Exercise 7.2 Evaluation and Management (Physical Examination)

1. Comprehensive examination: eight body systems were reviewed.

2. Expanded problem-focused examination: two systems reviewed (constitutional and integumentary)

3. Detailed examination. Note that the criteria for expanded problem focused and the detailed category are the same except in the level of specificity in the examination. Decisions on the level of specificity can be somewhat subjective.

Exercise 7.3 Evaluation and Management (Medical Decision Making)

1. Moderate complexity

[pic]2. Low complexity

[pic]3. Straightforward

[pic]

Exercise 7.4 Evaluation and Management Case Study

Established Patient History

[pic]Medical Decision Making

[pic]

1. Review the code descriptions for 99211–99215. Need two out of the three key components. The case study revealed a detailed history and comprehensive examination, and the medical decision making was of moderate complexity. The correct E/M code selection is 99214.

Exercise 7.5 Evaluation and Management

 1. True

 2. False

 3. False

 4. False

 5. False

 6. Extended—Four elements: yesterday (duration); getting ready to go to church (context); vomited bile (quality); deafness at the start (associated signs/symptoms)

 7. Expanded problem focused—Systems reviewed: respiratory, integumentary, ENT (ears, nose, throat), cardiovascular, gastrointestinal. Expanded problem focused (two to nine systems)

 8. Pertinent—One area (past history)

Exercise 7.6 Chapter 7 Review

 1. 99211

 2. 99203

 3. 99245

 4. 99202

 5. 99291

 6. 99281

 7. 99396

 8. 99213

 9. 99235

10. 99212

11. 99203

Chapter 8: Medicine

Exercise 8.1 Immunizations

1. 90460 Immunization administration, one vaccine/toxoid, with counseling

90461 x 3 (each additional vaccine/toxoid component)

90710 Vaccines, Measles, Mumps, Rubella and Varicella

2. 90471 Immunization administration, one vaccine/toxoid

90703 Vaccines, tetanus toxoid

3. 90471 Immunization administration, one vaccine/toxoid

90472 (add-on code) Immunization administration, each additional vaccine/toxoid

90632 Vaccines, hepatitis A

90658 Vaccines, influenza

4. 90470 Immunization administration, one vaccine/toxoid

90650 Vaccine, Human Papilloma Virus

5. 90471 Immunization administration

90472 each additional vaccine

90748 Vaccines, hepatitis B and Haemophilus influenzae B

Exercise 8.2 Psychiatry

1. 90885 Psychiatric diagnosis, evaluation of records or reports

2. 90847 Psychotherapy, family

3. 90834 Psychotherapy, Individual Patient/Family Member

Exercise 8.3 Dialysis

1. 90962 Dialysis, end-stage renal disease

2. 90945 Dialysis, peritoneal

3. 90969 Dialysis, end-stage renal disease

90969 (per day)

Exercise 8.4 Ophthalmology

1. 92018 Gonioscopy (See note under code 92020)

2. 92004 Ophthalmology, Diagnostic, Eye Exam, New patient

Exercise 8.5 Cardiovascular Services

1. 93025 Electrocardiography, Rhythm, Microvolt T-wave Alternans

2. 93452 Cardiac Catheterization, Left Heart, with Ventriculography

3. 92977 Thrombolysis, coronary vessels

4. 93015 Stress Tests, cardiovascular

5. 92924–LC Artery, coronary, atherectomy

6. 92920–LD Percutaneous Transluminal Angioplasty, artery, coronary

7. 93293 Telephone, Pacemaker Analysis

Exercise 8.6 Pulmonary Services

1. 94060 Spirometry, see Pulmonology, diagnostic, spirometry, evaluation

2. 94660 CPAP, see Continuous Positive Airway Pressure

3. 94450 Hypoxia, breathing response

Exercise 8.7 Allergy and Clinical Immunology

1. 95010 Allergy Tests, skin tests, venoms

2. 95120 Allergen Immunotherapy, allergen, prescription/supply/injection

Exercise 8.8 Injections and Infusions

1. 96372 Injection, intramuscular, therapeutic (Also, J3420 would be assigned for specific substance- B12.)

2. 96360 Infusion, Intravenous, Hydration

3. 96413 Chemotherapy, intravenous

96415 (add-on code)

Note: J code for Cisplatin would also be assigned.

4. 96420 Chemotherapy, intra-arterial

Exercise 8.9 Physical Medicine and Rehabilitation

1. 97032 x 2 TENS, see Physical Medicine, modalities, electric stimulation, attended, manual

2. 97113 x 2 Physical Medicine, aquatic therapy

3. 97006 Physical Medicine, athletic training, re-evaluation

4. 97602 Wound debridement, nonselective

Exercise 8.10 Chapter 8 Review

1. 90966 Dialysis, end-stage renal disease

2. 90471 Immunization administration, one vaccine/toxoid

90719 Vaccines, Diphtheria Toxoid

3. 93010 EKG, see electrocardiography, evaluation

4. 96413 Chemotherapy, intravenous

96415 x2

5. 93600 Electrophysiology Procedure

6. 92556 Audiometry, speech

7. 96372 Injection, intramuscular

99070 supply, materials

Note: If this were a Medicare patient, the J code to identify the substance (steroid) would be reported

8. 90832 Psychotherapy, Individual Patient/Family Member

Note: CPT provides guidance to choose the code closest to the actual time.

9. 92920 Atherectomies, coronary, see Artery, coronary, atherectomy

92921 (additional branch)

10. 92014–25 Ophthalmology, diagnostic, eye exam, established patient

92283 Opththalmology, diagnostic, color vision exam

11. 92920-RC Percutaneous Transluminal Angioplasty, artery, coronary

92973 Thrombectomy, percutaneous, coronary artery

92928-LC Coronary Artery, insertion, stent

12. 97761 x 3 Prosthesis training

13. 99050 Special services, after-hours medical services

Note: An E/M code would also be assigned in addition to this service.

14. 93925 Duplex scan, arterial studies, lower extremity

15. 95863 EMG, see electromyography, needle, extremities

16. 96920 Psoriasis Treatment

17. 98926 Manipulation, osteopathic

18. 99605 Medication Therapy Management

99607 (additional 15 minutes)

19. 99502 Home Services, newborn care

20. 94640 Inhalation Treatment, Pressurized or Nonpressurized

21. 95807 Sleep Study

Chapter 9: Anesthesia

Exercise 9.1 Chapter 9 Review

 1. 00406–P2 Anesthesia, breast

 2. 01730–P1 Anesthesia, arm, upper

99100

 3. 00567–P4 Anesthesia, heart, coronary artery bypass grafting

 4. 00540–P3 Anesthesia, lungs

 5. 00350–P1 Anesthesia, neck

 6. 00862-P2 Anesthesia, nephrectomy

 7. 00580–P5 Anesthesia, Heart, transplant

99100

99140

 8. 00350–P3 Anesthesia, neck

99100

 9. 00944–P1 Anesthesia, Hysterectomy, Vaginal

10. 00103-P1 Anesthesia, Eyelid

Chapter 10: HCPCS Level II

Exercise 10.1 HCPCS Level II Codes

1. A4611 Battery, heavy duty; ventilator

2. H1010 Education, family planning, nonmedical

3. J8520 Capecitabine, oral, 150 mg

4. M0076 Prolotherapy

5. Q0113 Pinworm Examination

Exercise 10.2 Chapter 10 Review

 1. E0196 Mattress, gel pressure

 2. A4550 Surgical, tray

 3. J3420 Vitamin, B12

 4. A4490 Surgical, stocking

 5. J1710 Hydrocortisone-sodium phosphate

 6. 51702 Insertion, catheter, urethra

A4355 Tubing set, Indwelling Foley catheter, each

 7. 29540 Strapping, ankle

E0112 Crutches

 8. 12011 Wound, repair, simple

Note: CMS instructs coders not to assign a modifier: “Do not use a modifier to indicate an anatomical site location on body (modifier 50 or Level II modifiers) if the narrative definition of a code indicates multiple occurrences.”

 9. 11730–TA Avulsion, nails

11732–T1

10. 26055–F8 Trigger finger repair

Chapter 11: Reimbursement in the Ambulatory Setting and Optional ICD-10-CM Coding Exercise

1. Incorrect. Because all repairs were of the same type (simple) and same site classification, the lacerations may be added together. The correct CPT code is 12014. Note that the use of modifiers is not appropriate with these codes.

ICD-10-CM Codes: S01.81xA (Laceration, forehead)

S01.112A (Laceration, left eyelid)

S01.111A (Laceration, right eyelid)

(an addition external cause code would also be assigned to the specific nature of the injury)

2. Incorrect. The CPT codebook provides codes 11200–11201 to report any type of removal of skin tags. The correct code is 11200.

ICD-10-CM Code: L91.8 (Tag, skin)

3. Incorrect. A diagnostic colonoscopy is included in a surgical colonoscopy; only the code for the surgical colonoscopy is assigned (45385).

ICD-10-CM Code: D12.0 (Polyp, cecum)

4. Incorrect. Code 52214 is in the urethra and bladder section and does not correctly identify the procedure. The correct code is 52354.

ICD-10-CM Code: N28.89 (Polyp, ureter)

5. Incorrect. Code 58720 is identified as a separate procedure and thus should not be used with 58150.

ICD-10-CM Code; D25.0 (Leiomyoma, uterus, submucous)

Note: Answers to exercises 6-10 are only in the Instructor’s Guide.

6. Incorrect. Code 80076 is an incorrect code. The test to measure alanine aminotransferase (ALT) (SGPT) was not performed. Total protein was not performed. The correct answer is 82040, 82247, 82248, 84075, 84450.

ICD-10-CM Code: R17 (Jaundice)

7. Incorrect. The D&C is included in the hysteroscopy code and does not warrant the additional code of 58120. NCCI lists 58558 as a comprehensive code and 58120 as a component.

ICD-10-CM Code: N93.8 (Hemorrhage, uterus, dysfunctional)

8. Correct.

ICD-9-CM Code: N63 (Mass, breast)

9. Incorrect. The lesion was incorrectly coded as benign. CPT code 11642 is the correct code.

ICD-10-CM Code: C44.319 (Carcinoma, basal cell—see Neoplasm, skin, chin, malignant, primary). There is a “see also” note for Neoplasm, skin, face and basal cell is indented under this entry. This code is difficult to find in the index, the final decision should be from the tabular list.

10. Incorrect. CPT code 31622 is a separate procedure code and should not be used with 31623. The only code needed is 31623.

ICD-10-CM Code: J20.9 (Bronchitis, acute)

................
................

In order to avoid copyright disputes, this page is only a partial summary.

Google Online Preview   Download